Будь умным!


У вас вопросы?
У нас ответы:) SamZan.net

Після тривалої роботи в гарячому цеху у робітника відмічено підвищення температури тіла до 3850С тахікар

Работа добавлена на сайт samzan.net: 2016-03-30

Поможем написать учебную работу

Если у вас возникли сложности с курсовой, контрольной, дипломной, рефератом, отчетом по практике, научно-исследовательской и любой другой работой - мы готовы помочь.

Предоплата всего

от 25%

Подписываем

договор

Выберите тип работы:

Скидка 25% при заказе до 20.5.2024

Патофізіологія серцево-судинної систе-

ми

1. Після тривалої роботи в гарячому цеху у робітника

відмічено підвищення температури тіла до 38,50С,

тахікардія, порушення ритму серцевих скорочень, АТ-

160/85 мм рт.ст. Втрата якого електроліту приводить

до порушення роботи серця?

А. *Калію

B. Кальцію

С. Магнію

D. Натрію

E. Фосфору

2. Зміни на ЕКГ проявлялися спочатку подовженням

інтервалу P-Q, потім випадінням поодиноких комплек-

сів QRS, пізніше – збільшенням кількості випадаючих

шлуночкових комплексів, і нарешті, передсердя скоро-

чувались з частотою 70 уд./ хв., а шлуночки – 35 уд./

хв. Описані зміни характерні для:

A. * Атріовентрикулярної блокади

B. Внутрішньопередсердної блокади

C. Внутрішньошлуночкової блокади

D. Екстрасистолії

E. Брадикардії

3. Хворий М., страждає хронічною серцевою недоста-

тністю. Який з перерахованих показників гемодинаміки

є головною ланкою патогенезу в розвитку серцевої

недостатності?

А. *Зменшення ударного об'єму

B. Розвиток тахікардії

С. Тоногенна дилятація

D. Збільшення периферичного опору судин

E. Підвищення центрального венозного тиску

4. При аналізі ЕКГ у хворого з інфарктом міокарду

була звернута увага на відсутність зубця Р, присут-

ність хвилеподібної ізоелектричної лінії з безліччю

дрібних зубців, часте і безладне розташування ком-

плексів QRS. Якому порушенню ритму відповідає

описана картина?

А. *Миготінню передсердь

B. Мерехтінню шлуночків

С. Ідіовентрикулярному ритму

D. Пароксизмальній тахікардії

E. Шлуночковій екстрасистолії

5. На ЕКГ під час дослідження хворого Р. виявлено

періодичну появу шлуночкової екстрасистоли. При

цьому встановлено, що передує екстрасистолії відсут-

ній зубець Р. Яка найвірогідніша причина його зник-

нення?

А. *Неможливість ретроградного проведення через

AV-вузол

B. Блокада проведення імпульсу по передсердю

С. Виникнення рефрактерного періоду в шлуночках

D. Блокада імпульсу в синусовому вузлі

E. Виникнення рефрактерного періоду в передсерді

6. У хворого Ш. з гіпертонічною хворобою при ультра-

звуковому дослідженні виявлена дилатація порожнин

серця. Яка з нижче перечислених ознак свідчить про

розвиток тоногенної дилатації?

А. *Розширення порожнин серця із збільшенням удар-

ного об'єму

B. Розширення порожнин без зміни ударного об'єму

С. Розширення порожнин із зменшенням ударного

об'єму

D. Рівномірне розширення меж серця

E. Нерівномірне розширення меж серця

7. У хворого В., 38 років при дослідженні ЕКГ знайшли

нерегулярні атріовентрикулярні екстрасистоли. Пору-

шення яких властивостей міокарду складає основу

патогенезу екстрасистолії?

А. *Збудливості

B. Автоматизму

С. Провідності

D. Скоротливості

E. Збудливості і провідності

8. У 48 літнього пацієнта після сильного психоемоцій-

ного навантаження раптово з'явився гострий біль в

ділянці серця з іррадіацією в ліву руку. Нітрогліцерин

зняв напад болю через 10 хвилин. Який патогенетич-

ний механізм є провідним в розвитку цього процесу?

А. *Спазм коронарних судин

B. Розширення периферичних судин

С. Закупорка коронарних судин

D. Здавлення коронарних судин

E. Підвищення потреб міокарду в кисні

9. Хворий, 44 років, скаржиться на задуху, серцебиття,

болі в правому міжребір’ї, набряки на ногах. На ЕКГ –

ознаки гіпертрофії обох шлуночків та правого перед-

сердя. Діагностовано недостатність тристулкового

клапана. Який патогенетичний варіант цієї недостат-

ності?

A. * Перевантаження серця об’ємом

B. Перевантаження серця опором

C. Первинна міокардіальна недостатність

D. Коронарна недостатність

E. Тампонада серця

10. При обстеженні у юнака 16 років було виявлено

прискорення серцебиття під час вдиху, сповільнення –

під час видиху. На ЕКГ відмічалося: вкорочення інтер-

валу RR під час вдиху та подовження його під час

видиху. Назвіть вид аритмії

A. * Дихальна аритмія

B. Миготлива аритмія

C. Синусова тахікардія

D. Ідіовентрикулярний ритм

E. Синусова брадикардія

11. Хворий А, 59 років, директор приватного підприєм-

ства. Після перевірки податковою інспекцією у нього

вечері з'явився інтенсивний пекучий біль, локалізова-

ний за грудиною, іррадіюючий в ліву руку. Через 15 хв.

стан хворого нормалізувався. Який з механізмів роз-

витку ішемії серцевого м'яза є провідним у даного

хворого?

А. *Підвищення в крові рівня катехоламінів

B. Атеросклероз коронарних артерій

С. Внутрішньосудинна агрегація форменних елементів

D. Здавлення коронарних артерій при дилатації поро-

жнин серця

E. Функціональне перевантаження серця

12. У хворої П., 45 років при електрокардіографічному

обстеженні на ЕКГ виявлено такі зміни: інтервал Р–Q

подовжений, при цьому випадає кожен другий або

третій комплекс QRST. Яке саме порушення провідно-

сті серця тут спостерігається?

A. *Атріовентрикулярна блокада ІІІ–го ступеня

B. Атріовентрикулярна блокада повна

C. Сино-атріальна блокада

D. Внутрішлуночкова блокада

E. Атріовентрикулярна блокада І–го ступеня

13. У хворого з гіпертонічною хворобою виявлено

значне збільшення маси міокарда лівого шлуночка. Це

сталося завдяки:

A. *Збільшення об’єму кардіоміоцитів

B. Збільшення кількості кардіоміоцитів

C. Розростання сполучної тканини

D. Затримки води в міокарді

E. Жирової інфільтрації міокарда

14. Електрик, працюючи з порушенням правил техніки

безпеки, випадково торкнувся оголеного електропро-

вода обома руками і загинув. Смерть настала внаслі-

док

A. * Фібриляції передсердь і шлуночків

B. Повної атріо-вентрикулярної блокади

C. Пригнічення автоматизму сино-атріального вузла

D. Зменшення скоротливої здатності міокарда

E. Порушення вагусної регуляції серця

15. Через 1 годину після накладання кільця, що звужує

аорту, в собаки різко зросла сила та частота серцевих

скорочень, а об’єм циркулюючої крові та товщина

стінки лівого шлуночка не відрізнялися від вихідних

показників. Яка стадія гіпертрофії міокарду спостеріга-

ється у тварини?

A. *Аварійна

B. Декомпенсації

C. Прогресуючого кардіосклерозу

D. Відносно стійкої гіперфункції

E. Завершеної гіпертрофії

16. У хворого на артеріальну гіпертензію наслідком

гіпертонічного кризу стала гостра серцева недостат-

ність. Який механізм серцевої недостатності є голо-

вним в даному випадку?

A. * Перевантаження серця опором

B. Перевантаження серця об'ємом крові

C. Ушкодження міокарда

D. Абсолютна коронарна недостатність

E. Відносна коронарна недостатність

17. При аналізі ЕКГ встановлено: ритм синусовий,

правильний, інтервал RR 0,58 сек, розташування і

тривалість інших інтервалів, зубців і сегментів не

змінені. Назвіть вид аритмії.

A. *Синусова тахікардія

B. Синусова брадикардія

C. Ідіовентрикулярний ритм

D. Синусова аритмія

E. Миготлива аритмія

18. При відтворенні артеріальної гіпертензії у собаки

через 1 місяць товщина стінки лівого шлуночка зросла

в 1.7 рази, а об’єм циркулюючої крові не змінився

порівняно з вихідними даними. Яка стадія гіпертрофії

міокарда спостерігається у тварини?

A. * Завершеної гіпертрофії

B. Аварійна

C. Початкова

D. Прогресуючого кардіосклерозу

E. Декомпенсації

19. Підліток 15 років, скаржиться на недостачу повітря,

загальну слабкість, серцебиття. ЧСС 130 уд/хв, АТ-

100/60 мм рт. ст. На ЕКГ, комплекс QRS нормальної

форми та тривалості. Число зубців Р та шлункових

комплексів однакове, зубець Т злитий з зубцем Р. Яка

аритмія серця спостерігається у підлітка ?

A. *Синусова тахікардія

B. Синусова екстрасистолія

C. Мерехтіння передсердь

D. Тремтіння передсердь

E. Передсердна пароксизмальна тахікардія

20. У чоловіка 45 років після значного психоемоційного

навантаження раптово з’явився стискаючий біль в

ділянці серця з іррадіацією в ліву руку, шию, під ліву

лопатку. Обличчя стало блідим, вкрилось холодним

потом. Нітрогліцерин зняв приступ болю. Який процес

розвився у хворого?

A. *Стенокардія

B. Інфаркт-міокарду

C. Інсульт

D. Психогенний шок

E. Перфорація виразки шлунку

21. У хворого М, 45 років, при аналізі ЕКГ встановлено:

ритм синусовий, число передсердних комплексів

більше числа шлуночкових комплексів; прогресуюче

подовження інтервалу P-Q від комплексу до комплек-

су; випадіння окремих шлуночкових комплексів; зубці

Р та комплекси QRST без змін. Назвіть тип порушення

серцевого ритму.

A. *Атріовентрикулярна блокада II ступеня

B. Сино-артріальна блокада

C. Атріовентрикулярна блокада I ступеня

D. Внутрішньопередсердна блокадаE. Повна атріовентрикулярна блокада

22. Людина отримала електротравму. При цьому

струм пройшов через серцевий м'яз. Які небезпечні

порушення в роботі серця можуть виникнути у цій

ситуації, що вимагають термінового втручання?

A.* Фібриляція шлуночків

B. Фібриляція передсердь

C. Екстрасистолія

D. Тахікардія

E. Брадикардія

23. Який із показників прямо вказує на кардіальну

форму недостатності кровообігу?

A.* Зменшення швидкості зростання тиску в лівому

шлуночку в фазу ізометричного скорочення

B. Підвищення мінімального артеріального тиску

C. Зниження максимального артеріального тиску

D. Тахікардія

E. Брадикардія

24.Електрокардіографічне дослідження пацієнта з

гіпертонічною хворобою показало такі результати:

ритм синусовий, правильний, частота серцевих скоро-

чень 92/хв, тривалість РQ – 0,2 с, QRS – не змінений.

У хворого має місце порушення:

A. *Автоматизму

B. Провідності

C. Збудливості

D. Рефрактерності

E. Скоротливості

25. Який із механізмів забезпечує термінову адаптацію

сили скорочення міокарду до коливань артеріального

тиску?

A. *Гомеометричний

B. Зміна симпатичного впливу на серце

C. Гормонально обумовлена зміна концентрації Na+ і

K+ в крові

D. Зміна парасимпатичного впливу на серце

E. Зміна інтенсивності виділення в кров гормонів Т3 і

Т4

26. Жінка 22 років. Діагностовано ревматичний міокар-

дит. Чим обумовлене порушення кровообігу?

A. *Пошкодженням міокарду

B. Зниженням об'єму циркулюючої крові

C. Перевантаженням міокарду збільшеним об'ємом

крові

D. Перевантаженням міокарду підвищеним опором

відтоку крові

E. Збільшенням об'єму судинного русла

27. Діагностовано стеноз аортального отвору. Чим

обумовлене порушення кровообігу?

A.* Перевантаженням міокарду підвищеним опором

відтоку крові

B. Пошкодженням міокарду

C. Зниженням об'єму циркулюючої крові

D. Перевантаженням міокарду збільшеним об'ємом

крові

E. Збільшенням об'єму судинного русла

28. Вказати коли має місце гетерометричний механізм

компенсації в лівому шлуночку:

A.*Недостатності аортального клапана

B. Стенозі аортального клапана

C. Недостатності клапана легеневої артерії

D. Стенозі правого атріовентрикулярного отвору

E. Стенозі мітрального клапана

29. Після перенесеного важкого інфекційного процесу

у хворого розвинувся міокардит з ушкодженням прові-

дної системи серця, що супроводжувалось періодич-

ною втратою свідомості внаслідок розвитку синдрому

Моргані - Адамса-Стокса. Яке порушення виникло у

хворого?

A. *Перехід неповної атріовентрикулярної блокади в

повну

B. Блокада лівої ніжки пучка Гіса

C. Блокада правої ніжки пучка Гіса

D. Пароксизмальна тахікардія

E. Передчасне збудження шлуночків

30. Дитина 10-ти років перенесла декілька атак ревма-

тизму. При клінічному обстеженні хлопчика було вста-

новлено, що мали місце запальні явища в суглобах і

виявилися ознаки недостатності мітрального клапану.

Яке з нижче перерахованих патологічних явищ у дано-

го хворого?

А.* Ревматизм

B. Артрит

С. Вада клапану

D. Запалення суглобів

E. Недостатність мітрального клапану

31. При обстеженні хворого встановлено, що артеріа-

льний тиск складає 190/100 мм.рт.ст. Який механізм

веде до підвищення артеріального тиску ?

А. *Спазм резистивних судин

B. Збільшення венозного тиску

С. Тоногенна дилатація серцевого м'яза

D. Міогенна дилатація серцевої м'яза

E. Зниження об'єму циркулюючої крові

32. У хворого з недостатністю мітрального клапану

виникла гіпертрофія лівого шлуночка серця. Який

механізм є пусковим у розвитку гіпертрофії?

A.*Активація генетичного апарату

B. Збільшення споживання жирних кислот

C. Збільшення інтенсивності клітинного дихання

D. Активація гліколізу

E. Збільшення надходження Ca2+ в клітину

33. У хворого виникла передсердно-шлуночкова бло-

када I ступеня. Які зміни ЕКГ можуть при цьому спо-

стерігатися?

A.* Продовження інтервалу PQ

B. Зниження висоти зубця R

C. Розширення комплексу QRS

D. Випадіння кожного 8-10 комплексу QRS

E. Розвиток ідіовентрикулярного ритму

34. У хворого 55 років діагностована гіпертонічна

хвороба. Який механізм є провідним в патогенезі

гіпертонічної хвороби?

A. *Підвищення периферичного опору судин

B. Збільшення частоти серцевих скорочень

C. Збільшення хвилинного об’єму крові

D. Збільшення маси циркулюючої крові

E. Веноконстрикція

35.У пацієнтки, яка перехворіла на грип, шляхом елек-

трокардіографічного дослідження виявлено наступне:

частота серцевих скорочень 140/хв, ритм синусовий,

коливання величини R-R не перевищують 0,15 с,

тривалість PQ – 0,2 c, QRS – не змінений. Ці показники

свідчать про розвиток:

A. *Синусової тахікардії

B. Синусової тахіаритмії

C. Фібриляції шлуночків

D. Пароксизмальної тахікардії

E. Мерехтіння шлуночків

36. З метою відтворення серцевої недостатності серце

жаби перфузували розчином бромистого кадмію -

блокатора сульфгідрильних груп. Який варіант серце-

вої недостатності при цьому виникає?

A.* Пошкодження міокарду внаслідок дефіциту енергії

B. Змішана форма

C. Від перевантаження

D. Пошкодження міокарду, що спричинене порушен-

ням вінцевого кровообігу

E. Пошкодження міокарду, що спричинене інтоксікаці-

єю

37. При аналізі ЕКГ встановлено: ритм синусовий,

число передсердних комплексів більше числа шлуноч-

кових комплексів; прогресуюче подовження інтервалу

P-Q від комплексу до комплексу; випадання окремих

шлуночкових комплексів, тому після зубця Р йде довга

пауза; зубці Р та комплекси QRST без змін. Назвіть

тип порушення серцевого ритму.

A.* Неповна атріо-вентрикулярна блокада ІІ ступеня

B. Сино-атріальна блокада

C. Неповна атріо-вентрикулярна блокада I ступеня

D. Неповна атріо-вентрикулярна блокада IIIступеня

E. Повна атріо-вентрикулярна блокада

38. У чоловіка 63 років зі слів родичів тричі відмічалась

втрата свідомості. ЧД 18 за 1 хв., ЧСС 45 за 1 хв., АТ

100/70 мм рт. ст. На ЕКГ: частота Р 80 за хвилину,

частота R 42 за хвилину, ритм правильний. Яка най-

більш вірогідна аритмія?

A. * Повна AV блокада

B. AV блокада II ступеню

C. AV блокада I ступеню

D. Синусова брадикардія

E. Сино-артріальна блокада

39. У хворого на ЕКГ знайдено зсув сегменту S-T вище

за ізоелектричну лінію на 1 мм і збільшення тривалості

зубця T до 0,25 С. З порушенням якого процесу пов'я-

зані вказані зміни на ЕКГ?

А. * Реполяризації шлуночків

B. Деполяризації шлуночків

С. Атріовентрикулярного проведення

D. Реполяризації передсердь

E. Деполяризації передсердь

40. Жінка 25 років скаржиться на постійний біль в

ділянці серця, задишку при русі, загальну слабість.

Об'єктивно: шкіра бліда та холодна, акроціаноз. Пульс

96 за 1 хв., АТ - 105/70 мм рт.ст. Межа серця зміщена

на 2 см вліво. Перший тон над верхівкою серця посла-

блений, систолічний шум над верхівкою. Діагностова-

но недостатність мітрального клапану серця. Чим

обумовлене порушення кровообігу?

A.* Перевантаженням міокарду збільшеним об'ємом

крові

B. Перевантаженням міокарду підвищеним опором

відтоку крові

C. Пошкодженням міокарду

D. Зниженням об'єму циркулюючої крові

E. Збільшенням об'єму судинного русла

41. Одним із найнебезпечніших моментів в патогенезі

некрозу міокарду є подальше наростання зон некрозу,

дистрофії та ішемії. Важлива роль в цьому належить

підвищенню споживання міокардом кисню. Які речови-

ни сприяють даному процесу?

A.* Катехоламіни

B. Ацетилхолін

C. Аденозин

D. Холестерин

E. Іони хлору

42. Хворий 59 років госпіталізований в кардіологічне

відділення в тяжкому стані з діагнозом: гострий ін-

фаркт міокарду в ділянці задньої стінки лівого шлуноч-

ку та перегородки, початковий набряк легень. Який

первинний механізм, що викликає розвиток набряку

легень у пацієнта?

A. *Лівошлуночкова недостатність

B. Легенева венозна гіпертензія

C. Легенева артеріальна гіпертензія

D. Гіпоксемія

E. Зниження альвеолярно-капілярної дифузії кисню

43. У хворого відмічається ціаноз, збільшення печінки,

набряки на нижніх кінцівках як наслідок правошлуноч-

кової недостатності. Яка причина розвитку правошлу-

ночкової недостатності?

А. *Гіпертонія малого кола кровообігу

B. Функціональне шунтування в легенях

С. Збільшення кількості катехоламінів

D. Підвищення венозного тиску

E. Кардіогенний цироз печінки

44. Через 3 тижні після гострого інфаркту міокарда у

хворого з'явились болі в серці та суглобах, запалення

легень. Який механізм є провідним у розвитку постін-

фарктного синдрому Дресслера у даного хворого?

A. *Аутоімунне запалення

B. Вторинна інфекція

C. Iшемія міокарду

D. Тромбоз судин

E. Резорбція білків з некротизованої ділянки міокарду

45. На ЕКГ пацієнта мають місце такі зміни: зубець Р –

нормальний, інтервал Р-Q - вкорочений, шлуночковий

комплекс QRST - розширений, зубець R - двогорбий

або двофазний. Яка із форм аритмії має місце в дано-

го пацієнта?

A.* Синдром WPW (Вольфа-Паркінсона-Уайта)

B. Синдром Фредеріка (тремтіння передсердь)

C. Атріо-вентрикулярна блокада

D. Миготіння шлуночків

E. Миготлива аритмія

46. У хворого на ішемічну хворобу серця на грунті

атеросклерозу коронарних артерій після коронарог-

рафії розвився тромбоз передньої міжшлуночкової

вінцевої артерії. Який механізм в розвитку цього

ускладнення є найбільш суттєвим?

A. *Пошкодження ендотелію судинної стінки

B. Сповільнення плину крові

C. Підвищення концентрації коагулянтів крові

D. Зменшення вмісту антикоагулянтів крові

E. Зниження активності фібринолітичної системи

47. У жінки, яка протягом 15 років страждала вираже-

ною гіпертензією, останнім часом з’явилась задишка,

серцебиття, трохи знизився систолічний тиск. Який

головний механізм виникнення у хворої серцевої

недостатності?

A. *Перевантаження серця збільшеним опором викиду

крові

B. Перевантаження серця збільшеним об’ємом крові

C. Ушкодження міокарду

D. Порушення проведення імпульсу по міокарду

E. Порушення регуляції серцевої діяльності

48. Хворий 21 р. поступив в стаціонар з загостренням

хронічного тонзиліту. Скаржиться на слабкість, задуху

при помірному фізичному навантаженні. Температура

37,50С. ЧСС 110 за хв. ЕКГ: ритм синусовий, інтервал

PQ подовжений. Яка аритмія у хворого?__ 203

A. *Предсердно-шлуночкова блокада І ст

B. Предсердно-шлуночкова блокада ІІ ст

C. Внутрішньопредсердна блокада

D. Порушення внутрішньошлуночкової провідності

E. Предсердно-шлуночкова екстросистолія

49. Хворий на механічну жовтяницю поступив в лікар-

ню з ознаками холемічного синдрому. На ЕКГ виявле-

на аритмія. Яке порушення ритму серця найбільш

ймовірне у хворого?

A. *Синусова брадикардія

B. Синусова тахікардія

C. Передсердна екстрасистола

D. Шлуночкова екстрасистола

E. Атріовентрикулярна блокада

50. У хворого має місце підвищення опору відтоку

крові з лівого шлуночка, що призвело до включення

гомеометричного механізму компенсації. При якому із

перерахованих патологічних процесів може мати місце

цей механізм компенсації у лівому шлуночку серця?

A. *Стеноз аортального клапану

B. Недостатність аортального клапану

C. Мітральний стеноз

D. Артеріальна гіпотензія

E. Емболія легеневої артерії

51. У хворого на дифтерію розвився міокардит з про-

явами недостатності кровообігу. Яке з порушень гемо-

динаміки є характерним для даного хворого?

A. * Зниження максимального артеріального тиску

B. Зниження мінімального артеріального тиску

C. Підвищення максимального артеріального тиску

D. Зростання швидкості кровотоку

E. Зниження венозного тиску

52. Аналізуючи електрокардіограму хворого, лікар

виявив на ній при частоті серцевих скорочень 60 уда-

рів за хвилину, періодично спостерігається укорочення

інтервалу діастоли Т- Р. Для якої аритмії характерна

дана зміна на ЕКГ?

А. *Для синусної екстрасистоли

B. Для предсердної екстрасистоли

С. Для предсердно-шлуночкової екстрасистоли

D. Для шлуночкової екстрасистоли

E. Для пароксизмальної тахікардії

53. Після перев’язки однієї з гілок вінцевих артерій у

собаки розвився інфаркт міокарду, який супроводжу-

вався проявами резорбційно-некротичного синдрому.

Назвіть найбільш характерну ознаку розвитку цього

синдрому.

A. *Збільшення в крові креатинкінази

B. Підвищення в крові катехоламінів

C. Біль за грудиною

D. Фібриляція шлуночків

E. Зниження хвилинного об’єму крові

54. У хворого з інфарктом міокарду артеріальний тиск

знизився до 70/40 мм рт. ст. Що є первинним механіз-

мом розвитку артеріальної гіпотензії при недостатності

серця?

A. *Зменшення хвилинного об’єму крові

B. Втрата організмом натрію

C. Затримка калію в організмі

D. Генералізована вазодилятація

E. Вазоконстрикція периферичних судин

55. Підвищення внутрішньочерепного тиску у хворого з

церебральною гематомою обумовило надмірну актив-

ність блукаючого нерва (ваготонію) та зміну частоти

серцевих скорочень. Який вид аритмії серця виникає

при цьому?

A. *Синусова брадікардія

B. Синусова тахікардія

C. Шлуночкова екстрасистолія

D. Пароксизмальна тахікардія

E. Передсердно-шлуночкова блокада

56. Хвора 45 років скаржиться на задишку при невели-

кому фізичному навантажені, набряки на ногах, в

анамнезі часті ангіни, хворіє протягом двох років.

Діагностовано недостатність кровообігу. Який гемоди-

намічний показник декомпенсації серця спостерігаєть-

ся в даному випадку?

A. *Зменшення хвилинного об’єму серця

B. Зменшення об’єму циркулюючої крові

C. Зменшення венозного тиску

D. Підвищення артеріального тиску

E. Тахікардія

57. У хворого на пневмосклероз розвилась легенева

гіпертензія та правошлуночкова серцева недостат-

ність з асцитом та набряками. Який основний патоге-

нетичний механізм розвитку набряків у цього хворого?

A. *Збільшення гідростатичного тиску у венах

B. Збільшення онкотичного тиску тканин

C. Зменшення онкотичного тиску крові

D. Зменшення осмотичного тиску крові

E. Збільшення проникності судин

58. У підлітка після перенесеного інфекційного захво-

рювання з’явилася різко виражена аритмія з вкоро-

ченням інтервалу R-R під час вдиху і подовження його

під час видиху. Що лежить в основі даного виду арит-

мії?

A. *Коливання тонусу блукаючого нерва під час акту

дихання

B. Порушення фунції збудливості серця

C. Порушення скоротливої функції серця

D. Рефлекс Бейнбріджа

E. Порушення функції провідності серця

59. У тварини відтворили гостру недостатність мітра-

льного клапана. Серце пристосувалось включенням

гетерометричного механізму компенсації, в основі

якого лежить:

A. *Закон Франка-Старлінга

B. Компенсаторна гіпертрофія міокарда

C. Зниження утворення кальцій-тропонінових комплек-

сів

D. Посилення біосинтезу білка

E. Механізм Боудіча

60. Жінка 49 років звернулась до лікаря зі скаргами на

підвищену втомлюваність і появу задишки при фізич-

ному навантаженні. Аналіз ЕКГ показав: частота сер-

цевих скорочень 50 ударів за 1 хв; РQ – подовжений,

QRS – не змінений, кількість зубців Р переважає кіль-

кість комплексів QRS. Який вид аритмії у хворої?

A. *Атріовентрикулярна блокада

B. Екстрасистолія

C. Синусова брадикардія

D. Миготлива аритмія

E. Синоатріальна блокада

61. Після перенесеної психічної травми у пацієнтки

періодично відбувається підвищення артеріального

тиску, що супроводжується головним болем, серце-

биттям, загальною слабістю. Який механізм лежить в

основі гіпертензії у хворої?

A. *Підвищення тонусу артеріол

B. Збільшення маси циркулюючої крові

C. Зниження хвилинного об’єму крові

D. Тахікардія

E. Веноконстрикція

62. Дистрофічні зміни серцевого м’язу супроводжу-

ються розширенням порожнин серця, зниженням сили

серцевих скорочень, збільшується об’єм крові, що

залишається під час систоли в порожнині серця, пере-

повнюються вени. Все це прояви:

A. *Міогенної дилатації

B. Тоногенної дилатації

C. Аварійної стадії гіпертрофії міокарду

D. Стадії кардіосклерозу

E. Тампонади серця

63. У хворого Д, 60 років, розвився крупновогнищевий

інфаркт міокарду, що ускладнився набряком легень.

Які кардіогемодинамічні порушення сприяли розвитку

набряку легень?

А. * Гостра лівошлуночкова недостатність

B. Гостра правошлуночкова недостатність

С. Змішана серцева недостатність

D. Кардіогенний шок

E. Колапс

64. У хворого з хронічною серцевою недостатністю

виявлено підвищення гідростатичного тиску в нижній

порожнистій вені, що викликало розвиток патологічно-

го процесу, який має назву:

A. *Серцевий набряк

B. Печінковий набряк

C. Нирковий набряк

D. Лімфатичний набряк

E. Токсичний набряк

65. У хворого з гіпертонічним кризом виявлено в крові

збільшення концентрації ангіотензину ІІ. З чим

пов’язаний пресорний ефект ангіотензину?

A. *Скороченням м’язів артеріол

B. Активізацією синтезу біогенних амінів

C. Гіперпродукцією простагландинів

D. Стимуляцією утворення вазопресину

E. Активацією калікреїн-кінінової системи

66. У тварини з недостатністю аортального клапану

розвилась гіпертрофія лівого шлуночка серця. В окре-

мих його ділянках визначаються локальні контрактури.

Накопичення якої речовини в міокардіоцитах виклика-

ло контрактури?

A.* Кальцію

B. Калію

C. Молочної кислоти

D. Вуглекислого газу

E. Натрію

67. У хворого на ішемічну хворобу серця раптово

з’явився тяжкий приступ стенокардії. Про нього: об-

личчя бліде, шкіра волога, холодна, АТ 70/50 мм рт.

ст. екстрасистолія. Діагностовано інфаркт міокарду та

кардіогенний шок. Назвіть первинний ланцюг патоге-

незу.

A. *Зменшення хвилинного об’єму крові

B. Екстрасистолія

C. Токсемія

D. Больовий синдром

E. Гіпотензія

68. У хворого з кардіосклерозом спостерігалась арит-

мія з кількістю предсердних скорочень до 400 в 1 хв.

При цьому частота пульсу була меншою за частоту

серцевих скорочень. Порушення якої функції серцево-

го м’язу виявляється в даному випадку?

A. *Збудливості та провідності

B. Автоматизму

C. Збудливості

D. Скоротливості

E. Провідності

69. У хворого з серцевою недостатністю виникла

аритмія у вигляді генерації позачергових імпульсів в

пучку Гіса. Порушення якої функції серцевого м’язу

спостерігається в даному випадку?

A. *Збудливості

B. Автоматизму

C. Провідності

D. Збудливості та провідності

E. Скоротливості

70. У новонародженої дитини віком 1 місяць діагносто-

вано вроджену недостатність аортального клапану.

При цьому спостерігається перевантаження лівого

шлуночка. Який механізм забезпечить компенсацію

серця в даному випадку?

A.* Гетерометричний

B. Гомеометричний

C. Тахікардія

D. Гіпертрофія міокарду

Е. Хроно-інотропний

71. Хворий К., 50 років, страждає на гіпертонічну хво-

робу. Під час фізичного навантаження у нього

з’явились відчуття м’язової слабкості, нестачі повітря,

синюшність слизової оболонки губ, шкіри, обличчя,

дихання супроводжувалось відчутними на відстані

вологими хрипами. Який механізм лежить в основі

виникнення такого синдрому?

A. * Гостра лівошлуночкова недостатність

B. Хронічна правошлуночкова недостатність

C. Хронічна лівошлуночкова недостатність

D. Колапс

E. Тампонада серця

72. У хворого 43 років артеріальна гіпертензія є нас-

лідком помірного збільшення об’єму серця за хвилину і

загального периферичного опору. Вкажіть гемодина-

мічний варіант розвитку артеріальної гіпертензії в

даному випадку.

A. * Еукінетичний

B. Гіперкінетичний

C. Гіпокінетичний

D. Змішаний

73. У чоловіка 65 років на протязі 15 років була вира-

жена артеріальна гіпертензія. Останнім часом систолі-

чний тиск почав знижуватись, а діастолічний залишив-

ся підвищеним. Який гемодинамічний тип артеріальної

гіпертензії у хворого?

A. *Гіпокінетичний

B. Нормокінетичний

C. Гіперкінетичний

D. Еукінетичний

E. Змішаний

74. У жінки з ішемічною хворобою серця на ЕКГ кіль-

кість серцевих скорочень 230 в хвилину, зубець Р

деформований, шлуночкові комплекси без змін. Які

порушення серцевого ритму у хворої?

A. *Предсердна пароксизмальна тахікардія

B. Миготлива аритмія

C. Тремтіння предсердь

D. Фибриляція шлуночків

E. Шлуночкова екстрасистола

75. У чоловіка, віком 55 років, який протягом багатьох

років страждав на недостатність мітрального клапану, 204

виникла гостра серцева недостатність. Який патофізі-

ологічний варіант недостатності серця спостерігається

у цьому випадку?

A. *Перевантаження серця об'ємом

B. Гіпоксичне ушкодження серця

C. Коронарогенне ушкодження серця

D. Нейрогенне ушкодження серця

E. Гостра тампонада серця

76. В експерименті у адреналектомованої тварини

спостерігали значну затримку калію в організмі, що

обумовила гіперкаліємію. Яке порушення ритму серця

найбільш ймовірне у такої тварини?

A. *Синусова брадикардія

B. Синусова тахікардія

C. Предсердна екстрасистола

D. Шлуночкова екстрасистола

E. Предсердно-шлуночкова блокада

77. У хворого 56 років з серцевою недостатністю спо-

стерігається набряк стоп та гомілок, шкіра в місці

набряку бліда і холодна. Яка провідна ланка патогене-

зу набряку у хворого?

A. *Підвищення гідростатичного тиску в венулах

B. Зменшення онкотичного тиску в капілярах

C. Підвищення проникливості капілярів

D. Порушення лімфовідтоку

E. Позитивний водний баланс

78. У хлопчика 14 років, хворого на дифтерію, в період

кризу при різкому падінні температури на фоні тахіка-

рдії А/Т складає 70/50 мм рт. ст До якої форми пору-

шення судинного тонусу відноситься дане явище?

А. *Гостра гіпотензія

B. Нейрогенна гіпотензія

С. Хронічна гіпотензія

D. Вегето-судинна дистонія

E. Гіпотонічна хвороба

79. У хворого, який страждає гіпертонічною хворобою,

виявлено добові коливання загального периферичного

опору судин току крові. З переважною участю яких

судин це зв'язано?

А. *Артеріол

B. Аорти

С. Капілярів

D. Артеріоло-венулярних анастомозів

E. Вен

80. У чоловіка 25 років виявлена недостатність мітра-

льного клапану без порушення кровообігу. Який не-

гайний механізм забезпечує серцеву компенсацію?

A. *Гетерометричний

B. Гомеометричний

C. Міогенна дилятація

D. Зменшення маси серця

E. Посилення синтезу катехоламінів

81. У хворого на ішемічну хворобу серця відмічається

гіпертрофія міокарду, тахікардія, зниження хвилинного

об’єму крові. Який з механізмів є провідним у пошко-

дженні кардіоміоцитів в даному випадку?

А. *Пошкодження специфічних мембранних насосів

B. Збільшення числа адренорецепторів

С. Втрата Мg2+ кардіоміоцитами

D. Втрата Са2+ кардіоміоцитами

E. Дегідратація кардіоміоцитів

82. Під час запису ЕКГ людині злегка натиснули паль-

цем па очні яблука і продовжували запис. Виникла

брадікардія. Порушення якої властивості провідної

системи серця лежить в основі цього явища?

А. *Автоматизму

B. Збудливості

С. Скоротливості

D. Провідності

E. Збудливості і провідності

83. Хвора Р., 39 років. поступила в нирковий центр у

важкому прекоматозному стані унаслідок розвитку

хронічної ниркової недостатності. АТ - 190/120 мм.рт.

ст. Який первинний патогенетичний механізм вірогідно

зумовив розвиток гіпертензії у хворої?

А. *Збільшення секреції реніну

B. Збільшення секреції глюкокортикоїдів

С. Зменшення секреції альдостерону

D. Збільшення секреції катехоламінів

E. Збільшення концентрації ангіотензинів в крові

84. Хворий з ревматичним міокардитом періодично

відчуває перебої в роботі серця. На ЕКГ обстеженні

виявлено нерегулярну появу ідіовентрикулярних екст-

расистол. Який патогенетичний механізм зумовлює

виникнення компенсаторної паузи при цих екстрасис-

толах?

А. *Рефрактерність міокарду до сприйняття чергового

імпульсу

B. Затримка збудження в атріовентрикулярному вузлі

С. Ретроградне проведення збудження до передсердя

D. Пригнічення функції синусного вузла

E. Порушення скоротливої здатності міокарду

85. У хворого з хронічною серцевою недостатністю при

УЗ-дослідженні виявлена гіпертрофія лівого шлуночку.

Які зміни в гіпертрофованому серці характерні в даній

ситуації?

А. *Зменшення енергозабезпечення міокардіоцитів

B. Збільшення кількості капілярів в міокарді

С. Активація електричної активності провідної системи

серця

D. Збільшення нервових закінчень в міокарді

E. Покращення кровопостачання

86. У хворого В., після розвитку інфаркту міокарду

відбулося різке падіння систолічного АТ до 60

мм.рт.ст. з тахікардією 140 уд/хв., задишкою, втратою

свідомості. Який механізм є провідним в патогенезі

розвитку даного стану?

А. *Зменшення хвилинного об'єму крові

B. Підвищення збудливості міокарду продуктами нек-

ротичного розпаду

С. Зниження об'єму циркулюючої крові

D. Розвиток пароксизмальної тахікардії

E. Розвиток анафілактичної реакції на міокардіальні

білки

87. У хворого на М., з хронічною серцевою недостатні-

стю було проведене планове дослідження показників

кардіогемодинаміки. Які найвірогідніші показники

свідчать про розвиток декомпенсації серця?

А. *Міогенна дилятація

B. Розвиток тахікардії

С. Тоногенна дилятація

D.Розвиток брадикардії

E. Підвищення центрального венозного тиску

88. У хворого А. з обширним інфарктом міокарду

розвинулась серцева недостатність. Який патогенети-

чний механізм сприяв її розвитку?

А. *Зменшення маси функціонуючих міокардіоцитів

B. Перевантаження тиском

С. Перевантаження об'ємом

D. Гостра тампонада серця

E. Реперфузійне ураження міокарду

89. У хворого з тривалим нападом стенокардії провели

ензимодіагностику. Збільшення рівня вмісту в крові

якого ферменту є вирішальним для підтвердження

діагнозу “інфаркт міокарду” в перші 2-4 години його

розвитку?

А. *Креатинфосфокінази

B. Альдолази

С. Ліпопротєїнліпази

D. Аланінамінотрансферази

E. Ацетілхолінестерази

90. У хірурга С. після проведення тривалої операції

підвищився АТ до 140/110 мм.рт.ст. Які зміни гумора-

льної регуляції можуть бути причиною підвищення

артеріального тиску в даному випадку?

А. *Активация симпато-адреналової системи

B. Активація синтезу і виділення альдостерону

С. Активація ренін-ангіотензивної системи

D. Активація калікрєїн-кінінової системи

E. Гальмування симпато-адреналової системи

91. У хворого має місце недостатність мітрального

клапану, внаслідок чого відбувається перевантаження

серця кров’ю. Який механізм термінової компенсації є

головним при перевантаженні серця об’ємом?

A. *Гетерометричний

B. Гомеометричний

C. Хроноінотропний

D. Інотропна дія катехоламінів

E. Гіпертрофія міокарда

92. Ухворого із стійкою артеріальною гіпертензією при

проведенні ангіографії виявлено атеросклеротичне

ураження обох ниркових артерій. Який механізм роз-

витку артеріальної гіпертензії є первинним?

A. *Збільшення продукції реніну

B. Збільшення катехоламінів

C. Збільшення виділення альдостерону

D. Збільшення серцевого викиду

E. Збільшення секреції вазопресину

93. У хворого із порушенням серцевого ритму при

обстеженні на ЕКГ спостерігається: ЧСС 50/хв., ритм

синусовий, не правильний; інтервал PQ подовжений;

періодичне випадіння комплексу QRS. Яке порушення

серцевого ритму має місце?

A. *Неповна а/v блокада ІІ ст

B. Повна а/v блокада

C. Блокада правої ніжки пучка Гіса

D. Неповна а/v блокада І ст

E. Синдром слабості синусового вузла

94. Одним з механізмів ушкодження кардіоміоцитів при

ішемії міокарду є активація перекисного окислення

ліпідів. Цей процес стимулюється внаслідок підвищен-

ня вмісту в міокарді таких речовин:

A. * Катехоламінів

B. АТФ

C. Глутатіонпероксидази

D. Супероксиддисмутази

E. Вільних жирних кислот

95. У жінки 32 років після перенесеного міокардиту

виявлено при електрокардіографічному дослідженні

порушення серцевого ритму (ритм не синусний). Функ-

ції яких кардіоміоцитів порушені?

A. * Пейсмекерних клітин

B. Скоротливих кардіоміоцитів

C. Перехідних провідних кардіоміоцитів

D. Провідних кардіоміоцитів пучка Гіса

E. Провідних кардіоміоцитів ніжок пучка Гіса

96. В умовах гострого експерименту у собаки видали-

ли обидві нирки і перевели на гемодіаліз. Внаслідок

цього виникло стійке підвищення артеріального тиску.

Головним механізмом його виникнення є:

A. *Зниження депресорних простогландинів

B. Збільшення продукції альдостерону

C. Збільшення антидіуретичного гормону

D. Активація ренін – ангіотензивної системи

E. Збільшення виділення норадреналіну

97. У дитини 5 років на ЕКГ спостерігалось порушення

ритму серцевої діяльності. При затримці диханя ритм

серцевої діяльності ставав правильним. Який вид

порушень було зафіксовано?

A. *Дихальна аритмія

B. Миготлива аритмія

C. Синусна екстросистолія

D. Предсердна екстросистолія

E. Поперечна блокада серця

98. У чоловіка 50 років раптово виникло сильне серце-

биття, біль у серці, різка слабкість, підвищення АТ,

пульс неправильний з дефіцітом. На ЕКГ виявлено

відсутність зубця Р і різні інтервали R-R. Яке порушен-

ня серцевого ритму у хворого?

A. *Миготлива аритмія

B. Дихальна аритмія

C. Пароксизмальна тахікардія

D. Поперечна блокада серця

E. Синусова екстрасистолія

99. Чоловік 57 років скаржиться на біль в серці, який

виник після тривалих негативних емоцій. Лікар швид-

кої допомоги встановив ішемічну хворобу серця, що

проявилась стенокардією. Який механізм ішемії най-

більш вірогідний?

A. *Ангіоспастичний

B. Ангіоневротичний

C. Облітераційний

D. Компресійний

E. Обтураційний

100. При аналізі ЕКГ виявлено: зубці Р відсутні, за-

мість них чисельні хвилі f, комплекси QRST ідуть через

різні проміжки часу (RR неоднакові), зубці R різного

вольтажу. Назвіть вид аритмії.

A. * Миготлива аритмія

B. Синусова аритмія

C. Пароксизмальна тахікардія

D. Синусова тахікардія

E. Ідіовентрикулярний ритм

101. У хворого 65 років періодичні больові відчуття під

лівою лопаткою і в лівому плечі різко посилилися після

значного фізичного навантаження. Методом ЕКГ діаг-

ностовано гострий інфаркт міокарда. Який вид болю

мав місце в даному випадку?

A. *Іррадіюючий

B. Вісцеральний

C. Фантомний

D. Каузалгія

E. Міалгія

102. У хворого з серцевою недостатністю виникла

аритмія, при якій на ЕКГ частота скорочень перед-

сердь була 70, а шлуночків – 35 скорочень за 1 хв.

Порушення якої функції серцевого м’язу спостеріга-

ється у хворого?

A.* Провідності

B. Збудливості

C. Автоматизму205

D. Збудливості та провідності

E. Скоротливості

103. Хворий помер від інфаркту міокарду. Проведене

патогістологічне дослідження міокарду виявило значні

контрактурні зміни в кардіоміоцитах. Це зумовлено

нагромадженням в кардіоміоцитах іонів:

A. * Кальцію

B. Водню

C. Натрію

D. Магнію

E. Хлору

104. Яка експериментальна модель гіпертензії най-

більш відповідає сучасним уявленням про патогенез

есенціальної гіпертензії?

A. *Тварини із спадковою гіпертензією

B. Ренопривна

C. Рефлексогенна

D. Реноваскулярна

E. Кортикоїдна

105. Тромбоз коронарної артерії спричинив розвиток

інфаркту міокарду. Які механізми ушкодження клітин є

домінуючими?

А. *Ацидотичні

В. Кальцієві

С. Ліпідні

D. Електролітно-осмотичні

E. Протеїнові

106. У дівчини 15 років виявлено стеноз аорти, проте

розладів кровообігу у неї не спостерігалось. Який

негайний механізм забезпечує серцеву компенсацію?

A. *Гомеометричний

B. Підвищення АТ

C. Гетерометричний

D. Зменшення маси серця

E. Міогенна дилятація

107. У хворого має місце передсердно-шлуночкова

блокада І ступеня, що супроводжується подовженням

інтервалу P-Q до 0,25 с. Порушення якої функції серця

має місце за цих умов?

A. *Провідності

B. Автоматизму

C. Збудливості

D. Скоротливості

E. Засвоєння ритму

108. Трансмуральний інфаркт міокарду у хворого

ускладнився розвитком гострої лівошлуночкової недо-

статності серця. Що є найбільш типовим для цього

стану?

A. *Набряк легень

B. Набряк кінцівок

C. Ціаноз

D. Асцит

E. Артеріальна гіпертензія

109. У хворого після ішемії міокарду розвинувся репе-

рфузійний синдром. Зростання вмісту якого електролі-

ту в цитоплазмі кардіоміоцитів викликає посилення

ушкодження серця?

A. *Іонів кальцію

B. Іонів калію

C. Іонів магнію

D. Іонів хлору

E. Іонів натрію

110. У хворого виявлено екстрасистолію. На ЕКГ

відсутній зубець Р, комплекс QRS деформований, є

повна компенсаторна пауза. Які це екстрасистоли?

A.* Шлуночкові

B. Передсердні

C. Передсердно-шлуночкові

D. Синусні

111. У хворого, який скаржився на біль в ділянці лівої

лопатки, було діагностовано інфаркт міокарду. Назвіть

вид болю у хворого?

A. *Іррадіюючий (відбитий)

B. Вісцеральний

C. Фантомний

D. Перший (протопатичний)

E. Другий (епікритичний)

112. У пацієнта, який півтори місяці тому переніс ін-

фаркт міокарду, діагностовано синдром Дреслера з

характерною тріадою: перикардит, плеврит, пневмо-

нія. Який головний механізм цього ускладнення?

A. *Сенсибілізація організму антигенами міокарду

B. Зниження резистентності до інфекційних агентів

C. Активація сапрофітної мікрофлори

D. Інтоксикація організму продуктами некрозу

E. Викид у кров міокардіальних ферментів

113. У хворого діагностовано інфаркт міокарду. Визна-

чення рівня якого ферменту в крові є вирішальним для

проведення дифдіагностики в перші 2-4 години після

інфаркту?

A.* Креатинфосфокінази

B. Альдолази

C. Ацетилхолінестерази

D. Аланінамінотрансферази

E. Ліпопротеїдліпази

114. У хворого на ЕКГ виявлені поодинокі комплекси

QRST, перед якими діастоличний інтервал скороче-

ний, а після них трішки збільшений. Інтервал QRS цих

косплексів без змін, але вершина зубця R у всіх стан-

дартних відвіденнях в порівнянні з нормальними ком-

плексами дещо менша, а глибина зубця S більша.

Зубці P перед цими комплексами відсутні. Який це вид

аритмії?

A *Передсердно-шлуночкова екстрасистолія

B Передсердна екстрасистолія

C Шлуночкова екстрасистолія

D Передсердна блокада

E Передсердна шлуночкова блокада

115. У чоловіка, віком 35 років, під час тривалого бігу

виникла гостра серцева недостатність. Які зміни

іонного складу спостерігаються у серцевому м’язіпри

цьому стані

A *Накопичення в клітинах міокарду іонів Na+ і

Ca2+

B Накопичення в клітинах міокарду іонів K+ і

Mg2+

C Зменшення в клітинах міокарду іонів Na+ і

Ca2+

D Зменшення в позаклітинному просторі іонів

K+ і Mg2+

E Збільшення в позаклітинному просторі іонів

Na+ і Ca2+

116. У хворого, що знаходиться в кардіологіч-

ному відділенні з приводу серцевої недостатності,

виявлені зміни показників гемодинаміки. Який з них

найбільш інформативний для підтвердження вказаної

патології?

A *Зниження хвилинного об'єму крові

B Підвищення частоти серцевих скорочень

C Підвищення систолічного АТ

D Підвищення діастолічного АТ

E Підвищення венозного тиску

117. У хворого розвинувся напад загрудинного болю з

іррадіацією в ліву руку. Прийом нітрогліцерину через 5

хвилин зняв боль. Який найбільш вірогідний патогене-

тичний механізм лежить в основі патології, що розви-

нулась?

A *Ангиоспастична ішемія міокарду

B Тромбоз коронарних судин

C Справжній стаз в коронарних судинах

D Нейротонічна артеріальна гіперемія

E Емболія коронарних судин

118. На ЕКГ виявлено збільшення інтервалу PQ. Що

може бути причиною цього явища?

A *Порушення провідності від передсердь до шлуночків

B Порушення автоматизму синусного вузла

C Порушення збудливості АВ-вузла

D Порушення провідності по пучку Гіса

E Порушення провідності по волокнах Пуркінє

119. У хворого із цирозом печінки відмічається стійка

артеріальна гіпотензія. (АТ 90/50 мм.рт.ст.). Чим обу-

мовлене зниження артеріального тиску при такій

патології печінки?

A * Зниженням синтезу ангіотензиногену

B Збільшенням синтезу Na-уретичного гормону

C Надмірною інактивацією вазопресину

D Посиленням рефлекторного впливу із рецепторної

зони дуги аорти

E Активацією калікреїн-кінінової системи

120. У хворого з вадою серця розвинулась хронічна

серцева недостатність. Одним з провідних патогене-

тичних механізмів її розвитку є порушення енергетич-

ного обміну. Яка речовина забезпечує транспорт

енергії АТФ з мітохондрій до місць його використання

в цитоплазмі?

A *Креатинфосфат

B НАДФН

C Коензим А

D Глутамін

E Транспортна РНК

121. У хворого швидко наростали симптоми

недостатності кровообігу. Лікарі швидкої допомоги

встановили гостру тампонаду серця. Ця патологія є

результатом:

A *Гострого ексудативного перикардиту

B Інфаркту міокарду

C Тромбозу легеневої артерії

D Фібриляції шлуночків

E Тромбозу лівого атріо – вентрикулярного отвору

122. При гострому інфаркті міокарду виникає декілька

зон в осередку інфаркту: зона некрозу, зона

ішемічного пошкодження і зона ішемії. Зоні пошкод-

ження на ЕКГ відповідає:

A *Зміщення сегменту RS-T вище ізолінії

B Негативний зубець Т

C Глибокий зубець Q

D Комплекс QRS типу QS

E Зниження зубця R

123. У хворого Н., 58 років, лікарі виявили трансму-

ральний інфаркт міокарду. Які зміни ЕКГ відповідають

зоні некрозу в осередку інфаркту?

A *Комплекс QRS типу QS

B Глибокий зубець Q

C Зміщення сегменту RS – T вище ізолінії

D Негативний зубець Т

E Зниження зубця R

124. У пацієнта з серцевою недостатністю періоди

дихання чергуються з періодами апное, які супровод-

жуються затьмаренням свідомості. Поява апное

пов’язана з:

A *Пригніченням нейронів дихального центру

B Пригніченням нейронів кори мозку

C Зниженням чутливості хеморецепторів судин

D Збудженням рецепторів дихальних шляхів

E Зниженням напруження СО2 у крові

125. У хворого 43 років з’явились ознаки інфаркту

міокарду, який ускладнився кардіогенним шоком.

Провідним механізмом в його розвитку стало:

A *Зниження скоротливої функції серця

B Зниження діастолічного наповнення серця

C Підвищення венозного тиску

D Збільшення частоти серцевих скорочень

E Підвищення периферичного опору

126. Хворий 49 років, водій за професією, скаржиться

на нестерпні стискаючі болі за грудиною, що

“віддають” у ділянку шиї, які виникли 2 години тому.

Стан важкий, блідість, тони серця послаблені. Лабора-

торне обстеження показало високий рівень

креатинкінази. Для якого захворювання характерні такі

симптоми?

A * Гострий інфаркт міокарду

B Гострий міокардит

C Стенокардія спокою

D Тромбоемболія легеневої артерії

E Нейро-циркуляторна дистонія

127. Чоловік 32 років скаржиться на загальну кволість,

спрагу, сухість у ротовій порожнині, нудоту. Останні

п’ять років спостерігається підвищення АТ. Лікувався

гіпотензивними засобами нерегулярно. Які зміни

гуморальної регуляції є причиною підвищення АТ?

A * Активація симптомато-адреналової системи

B Активація утворення і виділення альдостерону

C Інактивація ренин-ангеотензинової системи

D Активація калікреїн-кининової системи

E Активація утворення і виділення вазопресину

128. У хворого на гіпертонічну хворобу 50-річного віку

на тлі нападу гіпертонічного кризу розвинувся гострий

інфаркт міокарду. Для наданя невідкладної допомоги

лікарю потрібно вирішити, що з означеного нижче є

провідним для механізму розвитку інфарктів?

А.*Ішемія та наступна гіпоксія тканин, які оточують

ділянку інфаркту

В.Уповільнення зсідання і току крові, стрес

С.Загальний стрес і гіперкоагуляція

D.Гіпокоагуляція та прискорення току крові

Е.Руйнування форменних елементів крові

129. Гостра крововтрата викликала зниження систем-

ного АТ. Підвищення секреції якого гормону може

стабілізувати ситуацію, що склалась?

A * Реніну

B Кортикостерону

C Інсуліну

D Глюкагону

E Гастрину

130. У хворого на ЕКГ на кожні два зубці Р припадає

тільки один комплекс QRS, відстань Р-Р однакова,

відстань R-R також однакова й дорівнює сумі двох

інтервалів Р-Р. Порушення якої функції серцевого

м’яза спостерігається у хворого?

A*Провідності

B Збудливості206

C Автоматизму

D Автоматизму і збудливості

E Скорочення

131. На електрокардіограмі хворого з хронічною

ішемічною хворобою серця виявлено: ритм синусовий,

неправильний, частота скорочень 68 за хвилину, через

кожні 4 – 6 нормальних циклів зустрічаються вставні

екстрасистоли. Яка екстрасистола може бути встав-

ною?

A *Шлуночкова

B Синусна

C Передсердна

D З верхньої третини атріо-вентрикулярного вузла

E З середньої третини атріо-вентрикулярного вузла

132. В експерименті на щурах, при перев’язці вінцевої

артерії, був викликаний інфаркт міокарду. Який

механізм ушкодження кардіоміоцитів буде при цьому

провідним?

A * Ацидотичний

B Ліпідний

C Протеїновий

D Кальцієвий

E Нуклеїновий

133. У собаки шляхом часткового звуження просвіту

обох ниркових артерій відтворено хронічну

гіпертензію. Якою за патогенезом вона є?

A * Реноваскулярною

B Центрально-ішемічною

C Рефлексогенною

D Реноприпливною

E Мінералокортикоідною

134. У хворого діагностовано стеноз мітрального

клапану. До якого патофізіологічного типу

недостатності серця можна віднести цей процес?

A * Недостатність серця від перевантаженням опором

B Недостатність серця через ушкодження міокарду

C Недостатність серця через перевантаження

об’ємом

D Недостатність серця через гіпертрофію міокарду

E Змішана форма недостатності серця

135. У гострому експерименті на тварині викликали

ішемію міокарду, внаслідок чого зменшилася

амплітуда ПД у волокнах міокарду. Що спричинило

цей ефект?

A * Збільшення іонів K+ у міжклітинній рідині

B Збільшення іонів Na+ у міжклітинній рідині

C Зменшення іонів K+ у міжклітинній рідині

D Зменшення іонів Na+ у міжклітинній рідині

E Правильна відповідь відсутня

136. При проходженні профілактичного огляду у

чоловіка, який працює шахтарем, лікар встановив

зміни функціонального стану серця, що свідчать про

серцеву недостатність в стадії компенсації. Що з ниж-

че перечисленого є головним

підтвердженням компенсації діяльності серця?

A *Гіпертрофія міокарду

B Тахікардія

C Збільшення артеріального тиску

D Задишка

E Ціаноз

137. Хворий з ревматичним міокардитом періодично

відчуває перебої в роботі серця. При ЕКГ обстеженні

виявлена нерегулярна поява ідіовентрикулярних

екстрасистол. Який патогенетичний механізм зумов-

лює виникнення компенсаторної паузи при цих екстра-

систолах?

A *Рефрактерність міокарду до сприйняття чергового

імпульсу

B Затримка збудження в атріовентрикулярному вузлі

C Ретроградне проведення збудження до передсер-

дя

D Пригнічення функції синусового вузла

E Порушення скоротливої здатності міокарду

138. Хворий знаходиться в кардіологічному відділенні

з діагнозом: стеноз легеневої артерії. За час знахо-

дження в стаціонарі у хворого з'явився дифузний

ціаноз, збільшилася печінка, набрякли шийні вени,

з'явилися набряки на ногах. На ЕКГ - ознаки право-

шлуночкової недостатності. Який патогенетичний

механізм зумовив розвиток вказаних симптомів?

A *Підвищення венозного тиску у великому колі крово-

обігу

B Зниження швидкості скоротливого процесу в міокарді

C Порушення наповнення діастоли передсердя

D Дилатація порожнин серця

E Порушення електричної активності провідної системи

серця

139. У жінки 52 років з серцевою недостатністю, що

тривало приймає серцеві глікозиди на ЕКГ виявлено

порушення проведення імпульсів з передсердя до

шлуночків. Про що це свідчить?

A * Неповна АВ-блокада

B Блокада лівої ніжки пучка Гісса

C Блокада правої ніжки пучка Гісса

D Порушення проведення збудження по волокнах

Пуркіньє

E -

140. При обстеженні чоловіка 67 років на ЕКГ виявле-

но повне порушення провідності між передсердям і

шлуночками. Це:

A *Передсердя і шлуночки скорочуються незалежно

один від одного: передсердя у синусному ритмі, шлу-

ночки в ритмі пейсмекера

B Передсердя і шлуночки скорочуються в синусному

ритмі

C Передсердя і шлуночки скорочуються в ритмі

пейсмекера

D Передсердя і шлуночки скорочуються синхронно

E -

141. В експерименті у собаки перерізали правий блу-

каючий нерв. Які виникли порушення внаслідок цього?

A *Порушення автоматизму синусного вузла

B Порушення провідності в ділянці

атріовентрикулярного вузла

C Порушення провідності по передсердях

D Порушення провідності по пучку Гісса

E Порушення провідності по правій ніжці пучка Гісса

142. Артеріальна гіпертензія у хворого обумовлена

стенозом ниркових артерій. Активація якої системи є

головною ланкою в патогенезі цієї форми гіпертензії?

A. *Ренін-ангіотензинової

B. Симпато-адреналової

C. Парасимпатичної

D. Калікреїн-кінінової

E. Гіпоталамо-гіпофізарної

143. У пацієнта тривалість інтервалу P-Q на ЕКГ пере-

вищує норму при нормальній тривалості зубця P.

Причиною цього є зменшення швидкості проведення

збудження:

A. *Атріо-вентрикулярним вузлом

B. Волокнами Пуркін'є

C. Сино-атріальним вузлом

D. Ніжками пучка Гіса

E. Пучком Гіса

144. У чоловіка 35-ти років під час тривалого бігу

виникла гостра серцева недостатність. Які зміни

іонного складу спостерігаються у серцевому м'язі при

цьому стані?

A. *Накопичення в клітинах міокарда іонів Na+ і

Ca2+

B. Зменшення в позаклітинному просторі іонів K+

і Mg2+

C. Збільшення в позаклітинному просторі іонів

Na+ і Ca2+

D. Накопичення в клітинах міокарда іонів K+ і

Mg2+

E. Зменшення в клітинах міокарда іонів Na+ і

Ca2+

Патофізіологія зовнішнього дихання.

Гіпоксія

1. Чоловік 27 років, скаржиться на загальну слабкість,

задишку, підвищення температури (37-38°С). Захворів

через тиждень після переохолодження. Об'єктивно:

легкий ціаноз губ. ЧД 28 за хв., пульс 90 уд/хв. Права

половина грудної клітки відстає в акті дихання, міжре-

берні проміжки згладжені. При перкусії: нижче кута

правої лопатки тупість, дихання не прослуховується.

Діагноз: правосторонній ексудативний плеврит. Який

механізм розвитку недостатності дихання має місце у

хворого?

A. *Порушення екскурсії грудної клітини

B. Рестриктивна недостатність дихання

C. Обструктивна недостатність дихання

D. Порушення дифузії газів в легенях

E. Зменшення перфузії легень кров'ю

2. Хворий 38 років доставлений в приймальне відді-

лення з ознаками гіпоксії, що розвинулася після отру-

єння чадним газом. Об'єктивно: стан середньої тяжко-

сті, тахікардія, задишка, АТ 160/100. Який механізм

токсичної дії оксиду вуглецю на організм?

A. *Утворення карбоксигемоглобіну

B. Утворення метгемоглобіну

C. Порушення дисоціації оксигемоглобіну

D. Утворення карбгемоглобіну

E. Блокада кальцієвих каналів еритроцитів

3. У хворого П., з серцевою недостатністю виникли

явні ознаки гіпоксії головного мозку і розвилась задиш-

ка. В патогенезі якої задишки (з нижче перерахованих)

основною ланкою є зниження збудливості дихального

центру до вуглекислоти внаслідок кисневого голоду-

вання?

A. *Періодичне дихання

B. Експіраторна задишка

C. Інспіраторна задишка

D. Часте дихання

E. Глибоке дихання

4. Хворий Т., 19 років, з діагнозом “гостра пневмонія”

був направлений на лікування. Під час огляду: дихан-

ня 32 в хвилину, поверхневе. В дихальних рухах бе-

руть участь міжреберні м'язи. При аускультації вислу-

ховуються дрібноміхурцеві вологі і сухі хрипи. При

рентгеноскопії легень зміни, характерні для двосто-

ронньої крупозної пневмонії. Який механізм частого

поверхневого дихання в даному випадку?

A. *Подразнення легеневих рецепторів блукаючих

нервів

B. Інтоксикація продуктами запалення дихального

центру

C. Підвищення збудливості дихального центру

D. Гальмування кори мозку внаслідок інтоксикації

E. Гальмування блукаючих нервів

5. Хворий І., 38 років, поступив у відділення пульмоно-

логії з скаргами на біль в боці, що посилюється при

вдиху і кашлі. Больові відчуття зменшуються в лежа-

чому положенні на хворому боці. При обстеженні

дихання часте і поверхневе, помітно обмеження диха-

льних рухів відповідної половини грудної клітки. Який

механізм зміни характеру дихання у хворого?

A. *Обмеження корою мозку рефлексу Герінга-

Брейера

B. Надмірне подразнення легеневих рецепторів блу-

каючих нервів

C. Інтоксикація дихального центру продуктами запа-

лення

D. Підвищення збудливості дихального центру

E. Гальмування кори мозку внаслідок інтоксикації

6. Під час обіду дитина поперхнулась їдою. Почався

сильний кашель, шкіра і слизові цианотичні, пульс

прискорений, дихання рідке. Видих подовжений. Яке

порушення зовнішнього дихання розвилось у дитини?

A. *Стадія експіраторної задишки при асфіксії

B. Стадія інспіраторної задишки при асфіксії

C. Дихання Біота

D. Дихання Куссмауля

E. Дихання Чейна-Стокса

7. Чоловік 60 років внаслідок тривалого перебування в

мокрому одязі при низькій температурі навколишнього

середовища захворів крупозною пневмонією. Яка

причина виникнення такої форми запалення легень?

A. *Пневмокок

B. Вік

C. Зниження реактивності організму

D. Дія на організм низької температури

E. Дія на організм високої вологості

8. У чоловіка 48 років, хворого на хронічний бронхіт,

діагностована емфізема легень. Що є провідним чин-

ником даного ускладнення?

A. *Зниження еластичних властивостей легень

B. Зменшення розтягнення легень

C. Зменшення загальної течії крові в легенях

D. Порушення вентиляційно-перфузійного співвідно-

шення

E. Зменшення альвеолярної вентиляції

9. Хвора 23 років скаржиться на виражену слабкість,

сонливість, потемніння в очах, запаморочення, пере-

кручення смаку. В анамнезі меноррагії. Об'єктивно:

блідість шкірних покривів, тріщини в кутках рота, лам-

кість та розшарування нігтів, збільшення ЧД і ЧСС.

Аналіз крові: Ер 2,8 х 10¹²/л, Hb 70 г/л, КП 0,75. Яка

гіпоксія ймовірно привела до розвитку виявлених

симптомів у хворої?

A. *Гемічна

B. Циркуляторна

C. Тканинна

D. Респіраторна

E. Субстратна

10.У хворого виявлено порушення прохідності дихаль-

них шляхів на рівні дрібних і середніх бронхів. Які

порушення кислотно–лужної рівноваги можна виявити__Патофізіологія травлення та печінки

1. Хворий Л., скаржиться на відрижку, печію, часті

закрепи. При титруванні шлункового соку отримали

такі дані: загальна кислотність - 88 ммоль/л., загальна

HCl - 83 ммоль/л, вільна HCl - 50 ммоль/л, зв'язана

HCl - 33 ммоль/л, кислі фосфати і органічні кислоти - 5

ммоль/л. Оцінити стан кислотності шлунку:

A. *Гіперацидний стан

B. Гіпоацидний стан

C. Ахілія

D. Нормацидний стан

E. Гіпохлоргідрія

2. Хворому 25 років виставлено діагноз хронічного

гепатиту. Він скаржиться на втрату маси тіла на 10 кг

впродовж 2 місяців. Об'єктивно: відмічається сухість

та лущення шкіри, яка набула жовтуватого відтінку,

кровоточивість ясен. З порушенням якої функції печін-

ки пов”язані дрібноточкові крововиливи і кровоточи-

вість ясен?

A. *Білковосинтетичної

B. Пігментоутворюючої

C. Глікогенсинтетичної

D. Детоксікаційної

E. Депонуючої211

3. Хворий Ж, 48 років, знаходиться в реанімації після

отруєння чотирихлористим вуглецем. Стан важкий,

хворий без свідомості, пульс ниткоподібний, слабкого

наповнення, ЧСС 40 уд/хв. А/Т 75/40 мм.рт.ст., дихан-

ня періодичне типу Біота. У крові прямий білірубін -

155 мкмоль/л. У сечі високий вміст аміаку і білірубіну.

Що є основною причиною описаного стану у пацієнта?

A. *Печінкова недостатність

B. Кардіогенний шок

C. Дихальна недостатність

D. Серцева недостатність

E. Ниркова недостатність

4. У хворого 38 років, який переніс гепатит і продовжу-

вав вживати алкоголь, розвинулись ознаки цирозу

печінки з асцитом і набряками на нижніх кінцівках. Які

зміни складу крові стали вирішальними в розвитку

набряків?

A. * Гіпоальбумінемія

B. Гіпоглобулінемія

C. Гіпохолестеринемія

D. Гіпокаліємія

E. Гіпоглікемія

5. У хворого з синдромом Zollinger-Ellison (пухлина

підшлункової залози) відзначається збільшення сек-

реції, перистальтики шлунково-кишкового тракту, а

також діарея та пептичні виразки. Яка з перерахова-

них речовин, що секретується вказаною пухлиною,

викликає цей комплекс симптомів?

A. *Гастрин

B. Вазоактивний інтестинальний пептид

C. Пепсин

D. Трипсин

E. Секретин

6. Чоловік 20 років, що приймав участь у ліквідації

наслідків Чорнобильської катастрофи, захворів на

пародонтит. Який етіологічний чинник є найбільш

важливим в розвитку цієї патології?

A. * Емоційне перенапруження

B. Дефіцит заліза

C. Неповноцінне харчування

D. Підвищення фізичного навантаження на зубощеле-

пний апарат

E. Стрептококи порожнини рота

7. У тварини викликали карієс. Який компенсаторний

механізм є найважливішим при розвитку цього захво-

рювання?

A. * Утворення одонтобластами вторинного дентину

B. Новоутворення емалі

C. Гіпотрофія слинних залоз

D. Пригнічення фагоцитозу

E. Гіперфункція прищитовидних залоз

8. У хворого з запаленням трійчастого нерва останні-

ми роками прогресує пародонтит. Який чинник є голо-

вним у розвитку пародонтиту?

A. * Нейродистрофічні порушення у пародонті

B. Зменшення активності лейкоцитарної еластази

C. Зниження утворення імуноглобулінів

D. Підвищення тонусу вагуса

E. Зниження активності калікреїн-кінінової системи

9. У хворого, який хворіє жовчево - кам'яною хворо-

бою, внаслідок обтурації жовчовивідних шляхів вияв-

ляється знебарвлений жирний кал. Відсутність якого

компоненту жовчі приводить до стеатореї?

A. *Жовчних кислот

B. Холестерину

C. Жовчних пігментів

D. Жирних кислот

E. Лужної фосфатази

10. Чоловік, 43 років, після споживання жирної їжі та

алкоголю скаржиться на сильний біль у животі. У

сироватці крові вміст трипсину складає

850 ммоль/[год•л] [норма 60–240 ммоль/[год•л]]. Для

якої патології системи травлення це найбільш харак-

терно?

A*Гострий панкреатит

B Динамічна кишкова непрохідність

C Механічна кишкова непрохідність

D Виразка шлунка

E Гострий гастрит

11. У повної жінки 52 років встановлено цироз печінки.

Лабораторно: гіпоальбумінемія, гіперглобулінемія.

Візуально: набряк рук, повік, ніг. Найбільш імовірною

причиною набряків є:

A. *Зниження онкотичного тиску крові

B. Буферної емності крові

C. Кислотно-лужної рівноваги

D. Дезінтоксікаційної функції печінки

E. Глікогенсинтезуючої функції печінки

12. У хворого з алкогольним цирозом печінки скарги на

загальну слабкість, задишку. Встановлено зниження

артеріального тиску, асцит, розширення поверхневих

вен передньої стінки живота, спленомегалію. Яке

порушення гемодинаміки спостерігається у хворого?

A. *Синдром портальної гіпертензії

B. Недостатність лівого шлуночка серця

C. Недостатність правого шлуночка серця

D. Колапс

E. Тотальна серцева недостатність

13. До гастроентерологічного відділення потрапив

хворий 57 років з підозрою на синдром Золінгера-

Еллісона, про що свідчило різке збільшення рівня

гастрину у сироватці крові. Яке порушення секреторної

функції шлунку найбільш вірогідне в даному випадку?

A. * Гіперсекреція гіперацидна

B. Гіпосекреція гіперацидна

C. Ахілія

D. Гіпосекреція гіпоацидна

E. Гіперсекреція гіпоацидна

14. Чоловік 28 років, поступив у приймальний покій зі

скаргами на нудоту, блювоту, біль в правому підребе-

р'ї. Об'єктивно: жовтушність шкіри і склер, температу-

ра тіла підвищена, печінка збільшена, сеча темна, кал

гіпохолічний, гіпербілірубінемія (за рахунок прямого і

непрямого білірубіну), білірубінурія, уробілінурія,

гіпопротеїнемія, зниження зсідання крові. Для якого з

нижче перелічених станів найбільш характерні ці

зміни?

A.* Клітинно-паренхіматозної жовтяниці

B. Підпечінкової жовтяниці

C. Надпечінкової гемолітичної жовтяниці

D. Гострого холециститу

E. Гострого панкреатиту

15. Жінка 55 років, скаржиться на швидку втомлюва-

ність, дратівливість, безсоння та сонливість вдень,

свербіння шкіри. Пульс 58 уд/хв., АТ-110/65 мм рт.ст.

Кал безколірний, містить багато жиру. Підозрюється

жовчнокам'яна хвороба із обтурацією каменем жовчної

протоки. Що з переліченого нижче найбільш вірогідно

зумовлює стан хворої?

A.* Холемія

B. Гіпербілірубінемія

C. Гіперхолестеринемія

D. Порушення всмоктування жиророзчинних вітамінів

E. Порушення всмоктування жирів

16. Чоловік 32 років, звернувся до клінічного закладу з

приводу диспептичних розладів. При обстеженні знай-

дено виразковий дефект слизової оболонки шлунку та

діагностується синдром Золінгера-Еллісона. Що є

основним патогенетичним механізмом виникнення

виразки в даному випадку?

A.*Підвищення продукції гастрину

B. Підвищення продукції соляної кислоти

C. Зниження захисних властивостей слизової шлунку

D. Підвищення продукції інсуліну

E. Рефлюкс дуоденального вмісту у шлунок

17. Аналіз прозорої рідини лимонно-жовтого кольору,

отриманої у хворого при пункції черевної порожнини,

дав наступний результат: відносна щільність - 1012,

альбуміни - 1%, глобуліни - 0,2%, фібриногену немає,

еритроцити поодинокі, лейкоцити 1-3 в полі зору.

Рідина стерильна, при стоянні протягом години не

згортається. Яке з перерахованих клінічних проявів

супроводжується такими змінами?

A. *Асцит внаслідок застою крові в портальній системі

B. Запалення очеревини

C. Набрякова форма гемолітичної хвороби новонаро-

джених

D. Перитоніт

E. Емпієма очеревини

18. У хворого з жовтяницею при лабораторному дослі-

дженні отримані наступні дані, що характеризують

порушення пігментного обміну: загальний вміст у

сироватці крові білірубіну - 80 мкмоль/л; вміст прямого

білірубіну у сироватці крові -63 мкмоль/л; в сечі прису-

тній білірубін; уробіліноген і стеркобілін в сечі відсут-

ній. Яка причина відсутності в сечі уробіліногену при

даному патологічному процесі?

A.*Відсутність надходження білірубіну в кишечник

B. Порушення всмоктування уробіліну в кишечнику

C. Порушення виділення уробіліну нирками

D. Порушення захоплення уробіліну транспортним

білком

E. Дефіцит ферментів, перетворюючих глюкоронідбі-

лірубін в уробіліноген

19. Хворий Н. поступив в лікувальний заклад зі скар-

гами на диспептичні розлади, мелену, гемороїдальні

кровотечі. При дослідженні виявлено розширення сітки

венозних судин на передній стінці живота в комплексі

зі збільшенням його размірів. Яка патологія ШКТ може

проявлятись такими симптомами?

A.* Портальна гіпертензія

B. Кишкова аутоінтоксикація

C. Виразкова хвороба

D. Коліт

E. Ентерит

20. У хворого відмічається біль в епігастральній ділян-

ці, з іррадіацією в ліву лопатку. Виникнення болю

нерідко пов'язане з прийомом жирної їжі. Біль часто

супроводжується диспептичними розладами: відсутні-

стю апетиту, відразою до їжі, нудотою, блювотою,

метеоризмом. Стілець рясний /поліфекалії/, жирний

/стеаторея/, нерідко проноси. Яке захворювання мож-

на діагностувати в даному випадку?

A.* Хронічний панкреатит

B. Виразкова хвороба 12-палої кишки

C. Ентероколіт

D. Хронічне харчове отруєння

E. Хронічний гепатит

21. Хворий скаржиться на відрижку, печію, часті запо-

ри. При титруванні шлункового соку отримали такі

дані: загальна кислотність – 88ммоль/л, загальна HCl

– 83ммоль/л, вільна HCl – 50ммоль/л, зв'язана HCl –

33ммоль/л, кислі фосфати та органічні кислоти -

5ммоль/л. Оцініть стан кислотності шлункового соку.

A.* Гіперацидний

B. Гіпоацидний

C. Ахілія

D. Нормацидний

E. Гіперсекреція

22. Хворий скаржиться на біль в правому підребер'ї,

свербіння шкіри, головний біль, дратівливість, швидку

втомлюваність. При обстеженні встановлено: жовтуш-

ний колір шкіри та слизових оболонок, печінка збіль-

шена, болюча при пальпації. АТ - 80/40 мм рт.ст., ЧСС

- 46 за хвилину. В крові виявлено: вільний білірубін -

34,15 мкмоль/л, зв'язаний - 35,2 мкмоль/л, у сечі -

жовчні кислоти, зв'язаний білірубін, уробіліноген, вміст

стеркобіліногену в калі зменшений. Яка жовтяниця має

місце в даному випадку?

A.* Печінково-клітинна

B. Гемолітична

C. Підпечінкова

D. Обтураційна

E. Транспортна

23. Хворий дуже схуд внаслідок захворювання шлун-

ково-кишкового тракту, порушення процесів перетрав-

лювання та всмоктування. Які зміни білкових фракцій

слід очікувати при обстеженні?

A *Зниження альбумінів

B Підвищення альфа-глобулінів

C Підвищення бета-глобулінів

D Зниження глобулінів

E Підвищення гама-глобулінів

24. Хворий скаржиться на нудоту, відрижку, біль в

епігастральній ділянці, закрепи, схуднення. При об-

стеженні базальна секреція НСl - 6 ммоль/л, максима-

льна - 60 ммоль/л. Який стан кислотоутворюючої

функції в даного хворого?

A. *Гіперацидний

B. Гіпоацидний

C. Анацидний

D. Нормацидний

25. Хворий скаржиться на відрижку, печію, часті закре-

пи. При титруванні шлункового соку отримали такі

дані: загальна кислотність - 88 ммоль/л, загальна HCl -

83 ммоль/л, вільна HCl - 50 ммоль/л, зв'язана HCl - 33

ммоль/л, залишкова кислотність - 5 ммоль/л. Який

стан кислотоутворюючої функції шлунку?

A. *Гіперацидний

B. Гіпоацидний

C. Анацидний

D. Нормацидний

26. У хворого З., з вираженим жовтушним синдромом,

виявлено: у крові: рівень непрямого білірубіну - 34,5

мкмоль/л, прямого - 35,2 мкмоль/л; в сечі: жовчні

кислоти, уробіліноген; в калі – кількість стеркобіліноге-

ну зменшено. Який наибільш ймовірний вид жовтяниці

розвинувся у хворого?

A. *Печінкова

B. Надпечінкова

C. Підпечінкова__212

D. Гемолітична

Е. Обтураційна

27. У хворого зі скаргами на болі в епігастральній

ділянці оперізуючого характеру при лабораторному

обстеженні виявлено підвищений вміст діастази в сечі,

а також вміст у калі великої кількості неперетравлено-

го жиру. Для якої форми патології ШКТ характерні

описані явища?

A.* Гострий панкреатит

B. Виразкова хвороба шлунку

C. Гострий апендицит

D. Запалення товстого кишечника

E. Інфекційний гепатит

28. У дитячу поліклініку поступила 8-місячна дитина з

діареєю, здуттям живота, гіпотрофією, рясним зловон-

ним стільцем. Симптоми почали виявлятися і нароста-

ти після введення в раціон харчування борошняних

виробів. Стілець рясний, пінистий, безколірний з гни-

льним запахом, рН - 6. Яку патологію можна припусти-

ти в даному випадку?

A.* Целіакія

B. Гіполактазія

C. Панкреатит

D. Ентероколіт

E. Гіпоацидний гастрит

29. Через рік після субтотальної резекції шлунка з

приводу виразки малої кривизни виявлені зміни в

лабораторному аналізі крові: анемія, лейко- і тромбо-

цитопенія, кольоровий показник - 1,3, наявність мега-

лобластів та мегалоцитів. Дефіцит якого чинника

призводить до цих змін?

A.* Гастромукопротеїну

B. Хлористоводневої кислоти

C. Муцину

D. Пепсину

E. Гастрину

30. Дитина, 10-ти років, потрапила до лікарні з ознака-

ми гепатиту. При проведенні лабораторних

досліджень виявлено підвищену екскрецію міді з се-

чею. Для якої патології характерні вказані результати

дослідження?

A *Хвороба Вільсона

B Синдром Фанконі

C Цистінурія

D Хвороба Хартнупа

E Алкаптонурія

31. При гепатиті, інфаркті міокарда, в плазмі крові

хворих різко зростає активність аланін- і аспартатамі-

нотрасфераз. Які причини зростання активності цих

ферментів в крові?

A. *Пошкодження клітинних мембран з виходом фер-

ментів

B. Підвищення активності ферментів гормонами

C. Нестача піридоксину

D. Зростання швидкості синтезу амінокислот у ткани-

нах

E. Збільшення швидкості розпаду амінокислот у тка-

нинах

32. У чоловіка, віком 50 років, який лікувався на вираз-

кову хворобу шлунку, нормалізувалося травлення,

зникли болі, поліпшився настрій. Але через кілька

тижнів знову з’явилися болів епігастрії, печія, відриж-

ка кислим. Як можна характеризувати такий перебіг

хвороби?

A. *Рецидив хвороби

B. Період ремісії

C. Термінальний стан

D. Продромальний період

E. Латентний період

33. У жінки, віком 67 років, яка тривалий час стражда-

ла на холецистит, після їжі раптово виникла різка біль

у верхній частині живота, нудота, блювота. Встанов-

лено діагноз – гострий панкреатит. Що є основною

ланкою патогенезу цього захворювання?

A. *Передчасна активація ферментів підшлункової

залози

B. Зниження рівня ферментів у панкреатичному соці

C. Підвищення активації ферментів у дванадцятипалій

кишці

D. Зниження секреції панкреатичного поліпептиду

E. Підвищення рівня холецистокініну

34. Хвора, 48 років, поступила в клініку зі скаргами на

слабкість, дратівливість, порушення сну. Шкіра і скле-

ри жовтого кольору. В крові – прямий білірубін, холе-

мія; кал – ахолічний; сеча – темного кольору (біліру-

бін). Яка жовтяниця має місце у хворої?

A. * Механічна

B. Гемолітична

C. Паренхіматозна

D. Синдром Жільбера

E. Синдром Кріглера-Найяра

35. Хворого, 50 років, доправлено ургентно в лікарню з

підозрою на непрохідність верхніх відділів кишок. Які

зміни в організмі з перерахованих є

найхарактернішими для цього виду кишкової

непрохідності?

A *Надмірна втрата води

B Гіпоксія травлення

C Перитоніт

D Кишкова інтоксикація

E Біль

36. У хворого шлункова секреція підвищена як на

механічний, так і на хімічний подразник. Кислотність

шлункового соку висока. Після пробного сніданку

показник рН низький, і продовжує знижуватися протя-

гом наступних 2 годин. Який тип шлункової секреції

спостерігається у пацієнта?

A. *Збудливий

B. Гальмівний

C. Інертний

D. Астенічний

E. Нормальний

37. Хворий Л., 55 років, з гострим приступом печінкової

коліки поступив в гастроентерологічне відділення.

Об'єктивно: температура тіла 38°С , склери, слизові оболонки ішкіра іктеричні, сеча темна, кал світлий.

Скарги на шкірний зуд. Яка причина жовтяниці у дано-

го хворого?

A. * Обтурація жовчних шляхів

B. Деструкція гепатоцитів

C. Посилений розпад еритроцитів

D. Порушення ліпідного обміну

E. Тривалий прийом продуктів, багатих каротином

38. При обстежені хворого з виразковою хворобою

шлунку, встановлено, що шлункова секреція досягає

максимуму через 5 хвилин після прийому їжі, а зни-

ження її наступає через 1 годину 25 хвилин. Який тип

шлункової секреції властивий даному хворому?

A. *Збудливий

B. Нормальний

C. Гальмівний

D. Астенічний

E. Інертний

39. До лікаря звернувся пацієнт з приводу жовтушності

склер і шкіри. При обстеженні не було виявлено енце-

фалопатії та ознак холемічного і ахолічного синдромів.

Яка жовтяниця розвинулася в цього пацієнта?

A.* Гемолітична

B. Паренхіматозна

C. Ядерна

D. Механічна

E. Ензимопатична

40. У хворого кислотність шлункового соку складає 20

умовних одиниць. Що може спричинити таке зниження

кислотності шлункового соку?

A.*Хронічний атрофічний гастрит

B. Хронічний гіпертрофічний гастрит

C. Хронічний коліт

D. Виразкова хвороба дванадцятипалої кишки

E. Виразкова хвороба шлунка

41. У хворого, 42 років, скарги на болі в епігастральній

ділянці, блювоту; блювотні маси кольору “кофейної

гущі”; мелена. В анамнезі виразкова хвороба шлунку.

Аналіз крові: еритроцити – 2,8х10¹²/л, лейкоцити –

8х109/л, гемоглобін- 90 г/л. Вкажіть найбільш імовірне

ускладнення, яке виникло у хворого?

A. * Кровотеча

B. Пенетрація

C. Перфорація

D. Переродження в рак

E. Пілоростеноз

42. У хворого після отруєння грибами з’явилося жовте

забарвлення шкіри та склер, темний колір сечі. Який

пігмент спричиняє забарвлення сечі у хворого на

гемолітичну жовтяницю?

A. * Стеркобілін

B. Моноглюкуронід білірубіну

C. Некон’югований білірубін

D. Вердоглобін

E. Білівердин

43. Для моделювання виразки шлунка тварині ввели в

гастральні артерії атофан, який спричинює їх склеро-

зування. Який механізм пошкодження слизової оболо-

нки шлунку є провідним у даному експерименті?

A. * Гіпоксичний

B. Нейродистрофічний

C. Механічний

D. Дисрегуляційний

E. Нейрогуморальний

44. Хвора з хронічним гепатитом скаржиться на підви-

щення чутливості до барбітуратів, які раніше вона

переносила без симптомів інтоксикації. З порушенням

якої функції печінки це пов’язане?

A. * Метаболічної

B. Утворення жовчі

C. Гемодинамічної

D. Гемопоетичної

E. Фагоцитарної

45. У жінки, 57 років, після тривалого больового при-

ступу в правому підребер’ї з’явилася жовтяниця, після

чого хвора звернулася до лікаря. Виникла підозра на

наявність у хворої гострого калькульозного холецисти-

ту. Дослідження якого показника крові свідчить про

непрохідність жовчних протоків?

A. *Вільного та зв’язаного білірубіну

B. Білкових фракцій

C. Загальних ліпідів

D. Сечової кислоти

E. Залишкового азоту

46. Хлопчикові, 15 років, встановлено діагноз гострого

вірусного гепатиту. Дослідження якого показника крові

необхідно провести для підтвердження гострого ура-

ження печінкових клітин?

A. *Активність амінотрансфераз [АЛТ і АСТ]

B. Вміст вільного та зв’язаного білірубіну

C. Швидкість осідання еритроцитів [ШОЕ]

D. Рівень холестерину

E. Вміст білкових фракцій

47. Хворому поставили діагноз – синдром подразненої

кишки. Виділення якого інкрету найбільш вірогідно

збільшується при цьому?

A. *Мотиліну

B. Інтестинального пептиду

C. Глюкагону

D. Урогастрону

E. Секретину

48. Клінічне обстеження хворого дозволило встанови-

ти попередній діагноз- рак печінки. Наявність якого

білка в сироватці крові дозволить підтвердити діагноз?

A. *Альфа-фетопротеїну

B. Пропердіну

C. Парапротеїну

D. С-реактивного протеїну

E. Гама-глобуліну

49. У хворого з пухлинною обтурацією жовчовивідних

шляхів в складі калу міститься велика кількість жиру

(стеаторея). Відсутність якого компоненту жовчі має

місце в даному випадку?

A. *Жовчних кислот

B. Жирних кислот

C. Холестерину

D. Жовчних пігментів

E. Лужної фосфатази

50. Хворий скаржиться на часті нудоти, які нерідко

завершуються блювотою. Порушення якої із функцій

шлунку ймовірно запідозрити в даного хворого?

A. * Евакуаторної

B. Екскреторної

C. Всмоктувальної

D. Інкреторної

E. Секреторної

51. У хворого діагностовано асцит. На животі виступа-

ють крупні судини синюшного відтінку. Ознакою якої

гіпертензії є даний синдром?

A. * Портальної

B. Малого кола кровообігу

C. Есенціальної

D. Церебро-ішемічної

E. Ниркової

52. У хворого з'явилися жовтушність шкіри, склер та

слизових оболонок. У плазмі крові підвищений рівень

загального білірубіну, в калі стеркобіліну, в сечі

уробіліну. Який вид жовтяниці у хворого?

A *Гемолітична

B Хвороба Жільбера

C Паренхіматозна

D Обтураційна

E Холестатична

53. У хворого після перенесеного гепатиту розвинула-

ся печінкова недостатність. Порушення якої із функцій

печінки при цьому запускає механізм утворення__213

набряків?

A *Білковоутворюючої

B Бар’єрної

C Жовчоутворюючої

D Антитоксичної

E Глікогенутворюючої

54. Хворий тривалий час скаржиться на відчуття печії

в епігастральній ділянці і відрижку кислим. Порушення

якої функції шлунка найвірогідніше запідозрити у

хворого?

A *Секреторної

B Евакуаторної

C Резервуарної

D Моторної

E Інкреторної

55.У чоловіка, віком 50 років, у якого спостерігалася

гемолітична жовтяниця, було виявлено високий рівень

непрямого білірубіну в крові. В той же час у сечі цього

хворого непрямий білірубін був відсутній. Яка причина

зумовила виникнення цього явища?

A *Зв’язок білірубіну з альбуміном плазми

B Посилення екскреції білірубіну у дванадцятипалу

кишку

C Дія білірубіну на нейрони головного мозку

D Порушення холестеринового індексу жовчі

E Підвищення здатності жовчі до утворення каменів

56. У жінки, віком 40 років, у якої виникла механічна

жовтяниця, спостерігалось стрімке зростання рівня

холестерину в крові. Який механізм обумовив

гіперхолестеринемію у цієї хворої?

A *Підвищення надходження холестерину в кров у

складі жовчі

B Підвищення всмоктування холестерину в тонкому

кишечнику

C Підвищення естеріфікації холестерину в гепатоци-

тах

D Зниження утворення ліпопротеїдів низької і дуже

низької щільності

E Збільшення надходження холестерину до

гепатоцитів з крові

57. Чоловік, 65 років, страждає на рак печінки з син-

дромом портальної гіпертензії. Який вид портальної

гіпертензії має місце у хворого?

A. *Внутрішньопечінковий

B. Надпечінковий

C . Підпечінковий

D. Змішаний

E -.

58. В ургентну лікарню поступив хворий чоловік 50

років з сильним болем у животі, блювотою. При

обстеженні був встановлений діагноз: гострий пан-

креатит. Вкажіть головну ланку патогенезу у розвитку

панкреатиту.

A *Активація ферментів в протоках підшлункової зало-

зи

B Розвиток гіповолемії

C Білкове подразнення

D Метаболічний ацидоз

E Набряк підшлункової залози

59. Хворий 56 років скаржиться на слабкість, млявість,

порушення нічного сну, іктеричність слизових оболо-

нок та шкіри. При дослідженні виявлено збільшену

печінку та селезінку. Яке порушення виявили у хворо-

го?

A *Гепатолієнальний синдром

B Гепаторенальний синдром

C Регенераторна гіпертрофія печінки

D Синдром мальабсорбції

E Синдром портальної гіпертензії

60. Хворий, 50 років, був доправлений у лікарню

швидкою допомогою. При обстеженні: без свідомості,

рефлекси відсутні, широкі зіниці, жовтяниця,

геморагічний синдром, артеріальна гіпотензія,

печінковий запах з рота, дихання Куссмауля, змен-

шення розмірів печінки. Зі слів супроводжувачів –

зловживав алкогольними напоями. Який стан розви-

нувся у хворого?

A *Печінкова кома

B Ниркова кома

C Кетоацидотична кома

D Гіперглікемічна кома

E Церебральна кома

61. Хворий Т. доправлений в лікарню у важкому стані:

загальмованість, загальна слабкість, зниження АТ до

80/50 мм рт.ст. У анамнезі у хворого хронічний пан-

креатит внаслідок зловживання алкоголем. Погіршен-

ня стану стало наслідком вживання надмірно великого

одномоментного прийому алкоголю. Зниження артері-

ального тиску в даному випадку є результатом:

A *Впливу протеолітичних ферментів

B Серцевої недостатності

C Гальмування ЦНС

D Дегідратації організму

E Дії інсуліну

62. У хворої Т. після резекції шлунку на тлі інтенсивної

антибіотикотерапії з'явилися порушення з боку шлун-

ково-кишкового тракту: закрепи і здуття живота, зни-

ження апетиту. У хворого відмічена втрата ваги на 9

кг. При обстеженні встановлено зниження моторної

функції кишечника, стеаторея, неперетравлені м'язові

волокна в калі. У патогенезі цих порушень провідною

ланкою є:

A * Розлад мембранного травлення

B Порушення секреції підшлункової залози

C Порушення секреції жовчі

D Порушення секреції в шлунку

E Розлад порожнинного травлення

63. У хворого Н., який знаходиться на лікуванні з при-

воду вірусного гепатиту В з'явилися ознаки печінкової

недостатності. Які зміни крові, що свідчать про пору-

шення білкового обміну, найімовірніше будуть спосте-

рігатися в даному випадку?

A *Абсолютна гіпоальбумінемія

B Абсолютна гіперальбумінемія

C Абсолютна гіперфібриногенемія

D Білковий склад крові не змінений

E Абсолютна гіперглобулінемія

64. У хворого діагностована хвороба Жільбера, що

характеризується періодичною появою жовтяниці у

звязку з фізичним перевантаженням. При обстеженні,

жовчних пігментів у сечі нема, функціональні проби

печінки не порушені, печінка і селезінка не збільшені.

Яке порушення обміну жовчних пігментів спричинює

розвиток даної патології?

A * Спадкова відсутність рецепторів до білок-

білірубінового комплексу на поверхні гепатоцитів

B Порушення утворення диглюкуронід-білірубіну

C Порушення перетворення диглюкуронід-білірубіну

в уробіліноген

D Порушення відновлення уробіліногену до

стеркобіліногену

E Порушення виведення уробіліногену і

стеркобіліногену з сечею

65. У хворого після операції резекції шлунка розвинув-

ся демпінг-синдром, що проявляється раптовою

слабкістю, серцебиттям, посиленим потовиділенням,

тремтінням. Таке погіршення самопочуття з’являється

безпосередньо після або під час прийому їжі. Що

лежить в основі даної патології?

A *Швидке наповнення тонкої кишки їжею

B Відсутність шлункового соку

C Гіпоглікемія

D Посилення моторики кишечника

E Перерізка гілок блукаючого нерва

66. Пацієнт А., 56 років, після вживання жирної їжі з

алкоголем через деякий час відчув біль в епігастрії

зліва, який потім набув оперізуючого характеру.

Назвіть головну ланку патогенезу даної патології:

A *Активація протеолітичних ферментів в протоках і

клітинах підшлункової залози

B Порушення рівноваги між протеолітичними фермен-

тами та їх інгібіторами

C Підвищення тиску в панкреатичній протоці

D Порушення мікроциркуляції в підшлунковій залозі

E Посилення секреції панкреатичного соку

67. Хворий у непритомному стані доправлений швид-

кою допомогою у лікарню. Об’єктивно: рефлекси

відсутні, періодично з’являються судоми, дихання

нерівномірне. Після лабораторного обстеження було

діагностовано печінкову кому. Нагромадження якого

метаболіту є суттєвим для появи розладів центральної

нервової системи?

A *Аміаку

B Сечовини

C Глутаміну

D Білірубіну

E Гістаміну

68. Внаслідок переливання несумісної крові за антиге-

ном Rh у хворої виникла гемолітична жовтяниця. Який

лабораторний показник крові підтверджує цей тип

жовтяниці?

A *Нагромадження некон’югованого білірубіну

B Зменшення вмісту некон’югованого білірубіну

C Нагромадження уробіліногену

D Зменшення вмісту стеркобіліну

E Зменшення вмісту кон’югованого білірібіну

69. Хворому для покращення травлення жирної їжі

прзначено препарат жовчі. Які компоненти даного

препарату приймають участь в емульгуванні жирів?

A *Жовчні кислоти

B Холестерин і його ефіри

C Дигліцериди

D Білірубінглюкуроніди

E Вищі жирні кислоти

70. Пацієнт звернувся зі скаргами на гострий біль у

правому підребер’ї. При огляді лікар звернув увагу на

пожовтілі склери хворого. Лабораторні аналізи пока-

зали підвищену активність АЛТ та негативну реакцію

на стеркобілін в калі. Для якого захворювання

характерні такі симптоми?

A *Жовчнокам’яна хвороба

B Гемолітична жовтяниця

C Гепатит

D Хронічний коліт

E Хронічний гастрит

71. Чоловіка, 45-ти років, доставлено до лікарні з

діагнозом вірусний гепатит. Яких змін білків крові слід

очікувати у розпалі хвороби?

A *Зниження альбумінів та підвищення гамаглобулінів

B Різке підвищення кількості альфаглобулінів

C Зменшення альфа - і бетаглобулінів

D Підвищення альфа - і бетаглобулінів

E Підвищення альбумінів та зниження гамаглобулінів

72. У хворого виявили збільшення активності G-клітин

шлунку. Які з перерахованих змін травлення в шлунку

при цьому виникають?

A * Підвищена секреція гастрину

B Зменшення кислотності

C Зниження активності ферментів

D Утворення великої кількості слизу

E Пригнічення моторики шлунку

72. Тривале куріння призводить до зниження тонусу

стравохідного сфінктера. Які з перерахованих змін

процесів травлення можуть виникати при цьому?

A * Закидання їжі в стравохід

B Скупчення їжі в стравоході

C Накопичення їжі в шлунку

D Посилення моторики шлунку

E Прояв гастроінвагального рефлексу

73. Отруєння грибами супроводжувалося появою у

хворого жовтого забарвлення шкіри та симптомами

ураження ЦНС. Зі збільшенням в крові якої речовини

обумовлені порушення ЦНС при гемолітичній

жовтяниці?

A*Вільного некон’югованого білірубіну

B Некон’югованого білірубіну в комплексі з альбуміном

C Кон’югованого білірубіну

D Уробіліногену

E Жовчних кислот

74. У хворого на цироз печінки встановлено значне

зниження в крові вмісту альбумінів і глобулінів. Яке

порушення в організмі найбільш вірогідно може бути

наслідком такої гіпопротеїнемії?

A *Печінковий набряк

B Геморагічний синдром

C Анемія

D Енцефалопатія

E Аміноацидурія

75. У хворого з синдромом подразненої кишки в

слизовій оболонці порожньої кишки виявляють

підвищений вміст мотилину. Що є причиною розвитку

поносу в даному випадку?

A *Посилення перистальтики кишки

B Посилення гниття їжі в шлунку

C Гіперсекреція шлункового соку

D Гіперсекреція панкреатичного соку

E Дисбактеріоз

76. У новонародженої дитини на 5-6 добу з'явився

частий рідкий стілець, ознаки інтоксикації, розвинулось

зневоднення. Перехід на штучне годування кисло-

молочними сумішами привів до зникнення вказаних

явищ. Зроблено припущення про спадкову недостат-

ність лактози. Який процес при цьому був порушений у

дитини?

A. *Мембранного травлення

B. Порожнинного травлення

C. Екскреторної функції кишечника

D. Секреторної функції підшлункової залози

E. Секреції шлункового соку

77. У хворого А., 55 років, після резекції шлунку

з’явилися скарги на те, що кожного разу після прийому213

набряків?

A *Білковоутворюючої

B Бар’єрної

C Жовчоутворюючої

D Антитоксичної

E Глікогенутворюючої

54. Хворий тривалий час скаржиться на відчуття печії

в епігастральній ділянці і відрижку кислим. Порушення

якої функції шлунка найвірогідніше запідозрити у

хворого?

A *Секреторної

B Евакуаторної

C Резервуарної

D Моторної

E Інкреторної

55.У чоловіка, віком 50 років, у якого спостерігалася

гемолітична жовтяниця, було виявлено високий рівень

непрямого білірубіну в крові. В той же час у сечі цього

хворого непрямий білірубін був відсутній. Яка причина

зумовила виникнення цього явища?

A *Зв’язок білірубіну з альбуміном плазми

B Посилення екскреції білірубіну у дванадцятипалу

кишку

C Дія білірубіну на нейрони головного мозку

D Порушення холестеринового індексу жовчі

E Підвищення здатності жовчі до утворення каменів

56. У жінки, віком 40 років, у якої виникла механічна

жовтяниця, спостерігалось стрімке зростання рівня

холестерину в крові. Який механізм обумовив

гіперхолестеринемію у цієї хворої?

A *Підвищення надходження холестерину в кров у

складі жовчі

B Підвищення всмоктування холестерину в тонкому

кишечнику

C Підвищення естеріфікації холестерину в гепатоци-

тах

D Зниження утворення ліпопротеїдів низької і дуже

низької щільності

E Збільшення надходження холестерину до

гепатоцитів з крові

57. Чоловік, 65 років, страждає на рак печінки з син-

дромом портальної гіпертензії. Який вид портальної

гіпертензії має місце у хворого?

A. *Внутрішньопечінковий

B. Надпечінковий

C . Підпечінковий

D. Змішаний

E -.

58. В ургентну лікарню поступив хворий чоловік 50

років з сильним болем у животі, блювотою. При

обстеженні був встановлений діагноз: гострий пан-

креатит. Вкажіть головну ланку патогенезу у розвитку

панкреатиту.

A *Активація ферментів в протоках підшлункової зало-

зи

B Розвиток гіповолемії

C Білкове подразнення

D Метаболічний ацидоз

E Набряк підшлункової залози

59. Хворий 56 років скаржиться на слабкість, млявість,

порушення нічного сну, іктеричність слизових оболо-

нок та шкіри. При дослідженні виявлено збільшену

печінку та селезінку. Яке порушення виявили у хворо-

го?

A *Гепатолієнальний синдром

B Гепаторенальний синдром

C Регенераторна гіпертрофія печінки

D Синдром мальабсорбції

E Синдром портальної гіпертензії

60. Хворий, 50 років, був доправлений у лікарню

швидкою допомогою. При обстеженні: без свідомості,

рефлекси відсутні, широкі зіниці, жовтяниця,

геморагічний синдром, артеріальна гіпотензія,

печінковий запах з рота, дихання Куссмауля, змен-

шення розмірів печінки. Зі слів супроводжувачів –

зловживав алкогольними напоями. Який стан розви-

нувся у хворого?

A *Печінкова кома

B Ниркова кома

C Кетоацидотична кома

D Гіперглікемічна кома

E Церебральна кома

61. Хворий Т. доправлений в лікарню у важкому стані:

загальмованість, загальна слабкість, зниження АТ до

80/50 мм рт.ст. У анамнезі у хворого хронічний пан-

креатит внаслідок зловживання алкоголем. Погіршен-

ня стану стало наслідком вживання надмірно великого

одномоментного прийому алкоголю. Зниження артері-

ального тиску в даному випадку є результатом:

A *Впливу протеолітичних ферментів

B Серцевої недостатності

C Гальмування ЦНС

D Дегідратації організму

E Дії інсуліну

62. У хворої Т. після резекції шлунку на тлі інтенсивної

антибіотикотерапії з'явилися порушення з боку шлун-

ково-кишкового тракту: закрепи і здуття живота, зни-

ження апетиту. У хворого відмічена втрата ваги на 9

кг. При обстеженні встановлено зниження моторної

функції кишечника, стеаторея, неперетравлені м'язові

волокна в калі. У патогенезі цих порушень провідною

ланкою є:

A * Розлад мембранного травлення

B Порушення секреції підшлункової залози

C Порушення секреції жовчі

D Порушення секреції в шлунку

E Розлад порожнинного травлення

63. У хворого Н., який знаходиться на лікуванні з при-

воду вірусного гепатиту В з'явилися ознаки печінкової

недостатності. Які зміни крові, що свідчать про пору-

шення білкового обміну, найімовірніше будуть спосте-

рігатися в даному випадку?

A *Абсолютна гіпоальбумінемія

B Абсолютна гіперальбумінемія

C Абсолютна гіперфібриногенемія

D Білковий склад крові не змінений

E Абсолютна гіперглобулінемія

64. У хворого діагностована хвороба Жільбера, що

характеризується періодичною появою жовтяниці у

звязку з фізичним перевантаженням. При обстеженні,

жовчних пігментів у сечі нема, функціональні проби

печінки не порушені, печінка і селезінка не збільшені.

Яке порушення обміну жовчних пігментів спричинює

розвиток даної патології?

A * Спадкова відсутність рецепторів до білок-

білірубінового комплексу на поверхні гепатоцитів

B Порушення утворення диглюкуронід-білірубіну

C Порушення перетворення диглюкуронід-білірубіну

в уробіліноген

D Порушення відновлення уробіліногену до

стеркобіліногену

E Порушення виведення уробіліногену і

стеркобіліногену з сечею

65. У хворого після операції резекції шлунка розвинув-

ся демпінг-синдром, що проявляється раптовою

слабкістю, серцебиттям, посиленим потовиділенням,

тремтінням. Таке погіршення самопочуття з’являється

безпосередньо після або під час прийому їжі. Що

лежить в основі даної патології?

A *Швидке наповнення тонкої кишки їжею

B Відсутність шлункового соку

C Гіпоглікемія

D Посилення моторики кишечника

E Перерізка гілок блукаючого нерва

66. Пацієнт А., 56 років, після вживання жирної їжі з

алкоголем через деякий час відчув біль в епігастрії

зліва, який потім набув оперізуючого характеру.

Назвіть головну ланку патогенезу даної патології:

A *Активація протеолітичних ферментів в протоках і

клітинах підшлункової залози

B Порушення рівноваги між протеолітичними фермен-

тами та їх інгібіторами

C Підвищення тиску в панкреатичній протоці

D Порушення мікроциркуляції в підшлунковій залозі

E Посилення секреції панкреатичного соку

67. Хворий у непритомному стані доправлений швид-

кою допомогою у лікарню. Об’єктивно: рефлекси

відсутні, періодично з’являються судоми, дихання

нерівномірне. Після лабораторного обстеження було

діагностовано печінкову кому. Нагромадження якого

метаболіту є суттєвим для появи розладів центральної

нервової системи?

A *Аміаку

B Сечовини

C Глутаміну

D Білірубіну

E Гістаміну

68. Внаслідок переливання несумісної крові за антиге-

ном Rh у хворої виникла гемолітична жовтяниця. Який

лабораторний показник крові підтверджує цей тип

жовтяниці?

A *Нагромадження некон’югованого білірубіну

B Зменшення вмісту некон’югованого білірубіну

C Нагромадження уробіліногену

D Зменшення вмісту стеркобіліну

E Зменшення вмісту кон’югованого білірібіну

69. Хворому для покращення травлення жирної їжі

прзначено препарат жовчі. Які компоненти даного

препарату приймають участь в емульгуванні жирів?

A *Жовчні кислоти

B Холестерин і його ефіри

C Дигліцериди

D Білірубінглюкуроніди

E Вищі жирні кислоти

70. Пацієнт звернувся зі скаргами на гострий біль у

правому підребер’ї. При огляді лікар звернув увагу на

пожовтілі склери хворого. Лабораторні аналізи пока-

зали підвищену активність АЛТ та негативну реакцію

на стеркобілін в калі. Для якого захворювання

характерні такі симптоми?

A *Жовчнокам’яна хвороба

B Гемолітична жовтяниця

C Гепатит

D Хронічний коліт

E Хронічний гастрит

71. Чоловіка, 45-ти років, доставлено до лікарні з

діагнозом вірусний гепатит. Яких змін білків крові слід

очікувати у розпалі хвороби?

A *Зниження альбумінів та підвищення гамаглобулінів

B Різке підвищення кількості альфаглобулінів

C Зменшення альфа - і бетаглобулінів

D Підвищення альфа - і бетаглобулінів

E Підвищення альбумінів та зниження гамаглобулінів

72. У хворого виявили збільшення активності G-клітин

шлунку. Які з перерахованих змін травлення в шлунку

при цьому виникають?

A * Підвищена секреція гастрину

B Зменшення кислотності

C Зниження активності ферментів

D Утворення великої кількості слизу

E Пригнічення моторики шлунку

72. Тривале куріння призводить до зниження тонусу

стравохідного сфінктера. Які з перерахованих змін

процесів травлення можуть виникати при цьому?

A * Закидання їжі в стравохід

B Скупчення їжі в стравоході

C Накопичення їжі в шлунку

D Посилення моторики шлунку

E Прояв гастроінвагального рефлексу

73. Отруєння грибами супроводжувалося появою у

хворого жовтого забарвлення шкіри та симптомами

ураження ЦНС. Зі збільшенням в крові якої речовини

обумовлені порушення ЦНС при гемолітичній

жовтяниці?

A*Вільного некон’югованого білірубіну

B Некон’югованого білірубіну в комплексі з альбуміном

C Кон’югованого білірубіну

D Уробіліногену

E Жовчних кислот

74. У хворого на цироз печінки встановлено значне

зниження в крові вмісту альбумінів і глобулінів. Яке

порушення в організмі найбільш вірогідно може бути

наслідком такої гіпопротеїнемії?

A *Печінковий набряк

B Геморагічний синдром

C Анемія

D Енцефалопатія

E Аміноацидурія

75. У хворого з синдромом подразненої кишки в

слизовій оболонці порожньої кишки виявляють

підвищений вміст мотилину. Що є причиною розвитку

поносу в даному випадку?

A *Посилення перистальтики кишки

B Посилення гниття їжі в шлунку

C Гіперсекреція шлункового соку

D Гіперсекреція панкреатичного соку

E Дисбактеріоз

76. У новонародженої дитини на 5-6 добу з'явився

частий рідкий стілець, ознаки інтоксикації, розвинулось

зневоднення. Перехід на штучне годування кисло-

молочними сумішами привів до зникнення вказаних

явищ. Зроблено припущення про спадкову недостат-

ність лактози. Який процес при цьому був порушений у

дитини?

A. *Мембранного травлення

B. Порожнинного травлення

C. Екскреторної функції кишечника

D. Секреторної функції підшлункової залози

E. Секреції шлункового соку

77. У хворого А., 55 років, після резекції шлунку

з’явилися скарги на те, що кожного разу після прийому214

їжі, протягом 20-ти – 30-ти хвилин з’являються слаб-

кість, головокружіння, серцебиття, нудота, посилене

потовиділення, відчуття гарячки. Сукупність таких

симптомів характерна для:

A *Демпінг синдрому

B Синдрому голодування

C Синдрому зневоднення

D Синдрому мальдигезії

E Синдрому мальабсорбції

78. У чоловіка, віком 65 років, у якого виник цироз

печінки, спостерігалось значне зниження

артеріального тиску крові. Який механізм може обумо-

вити артеріальну гіпотензію у цьому випадку?

A *Зменшення синтезу ангіотензиногену в печінці

B Зменшення утворення жовчних кислот у печінці

C Зменшення синтезу сечовини в печінці

D Зменшення антитоксичної функції печінки

E Зменшення синтезу транспортних белків у печінці

79. В експерименті при моделюванні печінкової

патології у тварин виникали брадикардія, зниження

артеріального тиску, ознаки пригнічення нервової

системи. Для якої патології печінки характерна така

сукупність ознак?

A *Холемічний синдром

B Ахолічний синдром

C Синдром портальної гіпертензії

D Гепато-ренальний синдром

E Дисхолія

80. У хворого після отруєння грибами різко погіршився

загальний стан: спостерігається втрата свідомості,

випадання рефлексів. Порушення якої функції печінки

стало причиною даних змін?

A * Зниження антитоксичної

B Порушення фагоцитарної

C Підвищення метаболічної

D Зниження секреторної

E Порушення гемодинамічної

81. У хворого, 39 років, при фіброгастроскопічному

обстеженні в пілоричному відділі шлунка виявлена

виразка розміром 1,5х1 см. Проба на наявність

Helicobacter pylori позитивна. Роль бактеріального

обсіменіння слизової шлунка в патогенезі виразки

пов’язана з:

A *Порушенням слизового бар”єра

B Підвищенням секреції кислоти

C Підвищенням секреції ферментів

D Розвитком запалення

E Ушкодженням клітин слизової

82. При сонографії у хворого виявлено дифузний

цироз печінки. Вміст цукру в крові становить 3,6

ммоль/л. Причиною розвитку гіпоглікемії було знижен-

ня :

A* Глікогену в печінці

B Глікогенолізу

C Гліконеогенезу

D Гліколізу

E Засвоєння глюкози

83. Через порушення білково-синтезуючої функції

печінки у хворого з печінковою недостатністю поруше-

ний синтез прокоагулянтів, протромбіну, фібріногену.

Який з нижче перерахованих синдромів може

очікуватися у цього пацієнта?

A *Геморагічний

B Синдром портальної гіпертензії

C Гепатолієнальний синдром

D Синдром ахолії

E Синдром холемії

84. У хворого-алкоголіка цироз печінки. В останні

півроку появились варикозно розширені вени на

животі, спленомегалія, асцит (синдром портальної

гіпертензії). Яке ускладнення з перерахованих є

найбільш очікуваною причиною можливої смерті

пацієнта?

A *Кровотеча з варикозних вен шлунково-кишкового

тракту

B Гепато-лієнальний синдром

C Гіпопротеїнемія

D Печінкова енцефалопатія

E Прискорений гемоліз еритроцитів

85 У хворого з'явилися жовтушність шкіри, склер та

слизових оболонок. У плазмі крові підвищений рівень

загального білірубіну, в калі - рівень стеркобіліну, в

сечі - уробіліну. Який вид жовтяниці у хворого?

A. *Гемолітична

B. Хвороба Жільбера

C. Холестатична

D. Обтураційна

E. Паренхіматозна

86. У хворого на жовтяницю у крові підвищений вміст

прямого білірубіну та жовчних кислот; у сечі відсутній

стеркобіліноген. При якій жовтяниці можлива наявність

цих ознак?

A. *Підпечінкова

B. Надпечінкова

C. Гемолітична

D. Паренхіматозна

E. Печінкова

87. Після видалення у пацієнта 2/3 шлунка у крові

зменшився вміст гемоглобіну, кількість еритроцитів,

збільшилися розміри цих клітин крові. Дефіцит якого

вітаміну призводить до таких змін у крові?

A. *B12

B. P

C. PP

D. C

E. B6

Патофізіологія 2л ирокн

1. У хворого важка нефропатія з вираженим набряко-

вим синдромом, що ускладнила хронічну бронхоекта-

тичну хворобу. Лабораторні дослідження: протеїнурія,

циліндрурія, значне зниження вмісту білка в сироватці

крові, гіперліпемія, гіпокаліємія та інші відхилення. Що

виступає первинною і найбільш суттєвою ланкою в

патогенезі набряків у даного хворого?

A. *Зниження онкотичного тиску крові

B. Підвищення гідростатичного тиску крові

C. Підвищення тиску позаклітинної рідини

D. Блокада лімфовідтоку

E. Підвищення проникності мікросудин

2. У хворого через півтора тижні після важкої стрепто-

кокової ангіни виявилася набряклість, підвищився

артеріальний тиск. У сечі гематурія і помірна протеїну-

рія. У крові антистрептококові антитіла і зниження

компонентів комплементу. У мікросудинах яких струк-

тур найбільш ймовірна локалізація скупчень імунних

комплексів, що зумовили розвиток нефропатії?

A. *Клубочків

B. Пірамідах

C. Мисок

D. Сечоводів

E. Сечового міхура

3. При обстеженні хворого встановлено, що кліренс

ендогенного креатиніну після збору 24-х годинного

зразку сечі у нього становить 50 мл/хв (при нормі –

110-150 мл/хв). Про зниження якої функції свідчить

наявність такої ознаки?

A. *Клубочкової фільтрації

B. Канальцевої реабсорбції

C. Інкреторної функції нирок

D. Виведення з організму іонів

E. Виведення з організму сечової кислоти

4. Пацієнт 49 р. впродовж 17 років страждає хронічним

гломерулонефритом. Пульс 82 за хвилину. АТ 190/120

мм рт.ст. Що виступає первинним механізмом підви-

щення артеріального тиску у хворого?

A. *Підвищення загального периферичного опору.

B. Збільшення об’єму циркулюючої крові;

C. Підвищення тонусу венозних судин;

D. Збільшення ударного об’єму крові;

E. Збільшення хвилинного об’єму крові;

5. У жінки з первинним гіперпаратиреоїдизмом пері-

одично повторюються напади ниркової коліки. Ультра-

звукове обстеження показало наявність дрібних каме-

нів в нирках, найбільш імовірною причиною утворення

яких є:

A. *Гіперкальціємія

B. Гіперфосфатемія

C. Гіперхолестеринемія

D. Гіперурікемія

E. Гіперкаліемія

6. У хворого після автомобільної травми артеріальний

тиск 70/40 мм рт.ст. Хворий непритомний. У добу

виділяє близько 550 мл сечі. Періодично виникають

судоми, дихання за типом Куссмауля. Як називається

таке порушення функції нирок?

A. *Гостра ниркова недостатність

B. Гострий дифузний гломерулонефрит

C. Тубулопатія

D. Хронічна ниркова недостатність

E. Пієлонефрит

7. У хворого А. після травматичного шоку розвинулись

ознаки ниркової недостатності. Якими патогенетични-

ми механізмами зумовлено цей стан?

A. *Зниженням об’єму клубочкової фільтрації

B. Обтурацією канальців нирок

C. Пригніченням екскреції сечі в канальцях

D. Блокування відтоку сечі

E. Пошкодження клубочкового апарату нирок

8. Чоловік 30 років, скаржиться на слабкість, головний

біль, біль у попереку, набряки на обличчі. Місяць тому

перехворів ангіною. Пульс - 84 уд/хв, АТ - 175/100 мм.

рт. ст. У сечі - еритроцити 40-52 в полі зору, лейкоцити

- 1-2 в полі зору, білок - 4 г/л. Встановлено діагноз

гострого дифузного гломерулонефриту. Який основ-

ний механізм ушкодження нирок у хворого?

A. *Імунне пошкодження клубочків

B. Пошкодження канальців

C. Порушення гемодинаміки в нирках

D. Порушення уродинаміки

E. Пряме пошкодження клубочків мікроорганізмами

9. Чоловік 32 років, чотири роки страждає хронічним

гломерулонефритом з нефротичним синдромом.

Об’єктивно: виражені набряки на обличчі, поширення

набряків на нижні кінцівки та тулуб. Який із переліче-

них механізмів найбільш характерний для розвитку

набряків у чоловіка?

A. *Зниження онкотичного тиску крові

B. Підвищення гідростатичного тиску крові у капілярах

C. Підвищення онкотичного тиску тканинної рідини

D. Утруднення лімфовідтоку

E. Підвищення проникливості капілярів

10. У людини з хронічним гломерулонефритом нарос-

тає загальна слабість, різка тахікардія з періодичною

аритмією, загальмованість і сонливість. Яке зрушення

КЛР супроводжує наближення уремічної коми?

A. *Негазовий ацидоз

B. Негазовий метаболічний ацидоз

C. Газовий ацидоз

D. Газовий алкалоз

E. Негазовий алкалоз

11. У хворого М. 55р. виявили гіперплазію кори надни-

рників. АТ – 190/90 мм.рт.ст.; в крові - вміст глюкози –

20 ммоль/л, збільшення кортикотропіну; у сечі - глюко-

зурія. Спостерігається ожиріння, гірсутизм. Для якої

патології характерні виявлені клінічні ознаки?

A. * Хвороби Іценко-Кушинга.

B. Хвороби Аддісона.

C. Синдрому Іценка-Кушинга.

D. Адипозогенітальної дистрофії.

E. Хвороби Барракера- Сіммондса.

12. Пацієнт 64 років з гострою серцевою недостатніс-

тю, артеріальним тиском 80/60 мм.рт.ст, добовим

діурезом 530 мл. В крові збільшена концентрація

сечовини і креатиніну. Назвіть патогенетичний меха-

нізм розвитку азотемії і олігурії:

A. *Зменшення фільтраційного тиску в клубочках.

B. Спазм приносних артеріол клубочка

C. Збільшення вироблення вазопресину

D. Зменшення об'єму циркулюючої крові

E. Гіпернатріємія

13. У хворого 24 років через півтора тижні після важкої

стрептококової ангіни виник набряк обличчя, підвищи-

вся АТ. В сечі – гематурія, протеїнурія 1,2 г/л. У крові

виявлені антистрептококові антитіла і зниження ком-

понентів комплементу. У мікросудинах яких структур

найбільш ймовірна локалізація скупчень імунних ком-

плексів, що зумовили розвиток нефропатії?

A. *Клубочки

B. Піраміди

C. Проксимальний відділ канальців

D. Петля Генле

E. Низхідний відділ канальців

14. Чоловік 32 років впродовж 4 років хворіє на хроніч-

ний гломерулонефрит. Госпіталізований з ознаками

анасарки. АТ - 185/105 мм рт.ст. У крові: Hb - 110 г/л,

ер. - 2,6х1012г/л, лейк. - 9,5х109г/л, залишковий азот -

32 ммоль/л, загальний білок - 50 г/л. Яка зміна з най-

більшою вірогідністю вказує на гломерулонефрит з

нефротичним синдромом?

A. *Гіпопротеїнемія

B. Анемія

C. Лейкоцитоз

D. Артеріальна гіпертензія

E. Гіперазотемія

15. В експерименті з моделювання ниркової патології у

тварини отримали: набряки, високу протеїнурію, гіпо-

протеїнемію, диспротеїнемію, гіперліпідемію. Для якої

патології нирок характерна така сукупність ознак?

A. *Нефротичний синдром.__ 215

B. Гострий дифузний гломерулонефрит.

C. Пієлонефрит.

D. Гостра ниркова недостатність.

E. Хронічна ниркова недостатність.

16. У хворого, після отруєння сулемою, розвинулася

гостра ниркова недостатність, перебіг якої включав 4

стадії: перша-початкова, друга-олігоанурія, четверта-

одужання. Як називається третя стадія гострої нирко-

вої недостатності?

A. *Поліурична

B. Метаболічна

C. Гемодинамічна

D. Ішемічна

E. Патохімічна

17. У хворого виявлено порушення реабсорбції глюко-

зи в проксимальному відділі нефрону з розвитком

глюкозурії, при цьому в плазмі крові має місце гіпоглі-

кемія. Як називається це порушення?

A. *Ниркова глюкозурія

B. Цукровий діабет

C. Фосфатний нирковий діабет

D. Позаниркова глюкозурія

E. Галактоземія

18. У хворого має місце пошкодження нирок з розвит-

ком масивної протеїнурії, гіпопротеїнемії, набряків та

ретенційної гіперліпемії. Це порушення має назву:

A. *Нефротичний синдром

B. Гостра ниркова недостатність

C. Хронічна ниркова недостатність

D. Ниркова гіпертензія

E. Ізогіпостенурія

19. Хворий, 19 років, переніс ангіну. Через 2 тижні

звернувся до лікаря із скаргами на олігурію, змінений

колір сечі (“м’ясних помиїв”). Артеріальний тиск –

190/100мм.рт.ст. Що є пусковим механізмом розвитку

артеріальної гіпертензії в даному випадку?

A. * Ішемія ниркових клубочків.

B. Гіперсекреція альдостерону.

C. Гіперсекреція реніну.

D. Збільшення продукції АДГ.

E. Підвищення нейрогенного компоненту судинного

тонусу.

20. У 20-річного хлопця через 2 тижні після перенесе-

ної лакунарної ангіни з'явилися скарги на загальну

слабкість, набряки під очима. Після обстеження хво-

рому встановлено діагноз: гострий гломерулонефрит.

Які патологічні зміни у складі сечі найбільш вірогідні?

A. *Протеїнурія

B. Циліндрурія

C. Наявність свіжих еритроцитів

D. Піурія

E. Натрійурія

21. Внаслідок передозування вазодилятатора у паціє-

нта артеріальний тиск знизився до 60/40 мм.рт.ст. та

виник колапс. До якої патології це може привести?

A. *Гострої ниркової недостатності

B. Інфаркту міокарду

C. Гострої печінкової недостатності

D. Інсульту

E. Гострої дихальної недостатності

22. У фізично здорових молодих вояків після важкого

фізичного навантаження (одноденний піший перехід

на 50 км) в сечі виявлено білок, рівень якого в серед-

ньому не перевищував 1 г/л. Який різновид протеїнурії

мав місце?

A. *Маршова протеїнурія

B. Дегідратаційна протеїнурія

C. Аліментарна протеїнурія

D. Органічна протеїнурія

E. Несправжня протеїнурія

23. У хворого, внаслідок крововиливу у ділянку задньої

долі гіпофізу, виникла поліурія, зменшився рівень

вазопресину в крові. Що є головним механізмом роз-

витку поліурії в даному випадку?

A. *Зменшення реабсорбції води в ниркових канальцях

B. Збільшення фільтрації води в клубочках

C. Збільшення реабсорбції натрію в канальцях

D. Зменшення реабсорбції натрію в канальцях

E. Збільшення екскреції калію

24. У хворого М., діагностовано хронічний гломеруло-

нефрит. Внаслідок значних склеротичних змін маса

функціонуючих нефронів зменшилася до 10%. Яке з

перерахованих нижче порушень лежить в основі наро-

стаючого уремічного синдрому?

A. *Азотемія

B. Порушення водного гомеостазу

C. Порушення осмотичного гомеостазу

D. Ниркова остеодистрофія

E. Артеріальна гіпертензія

25. Хвора Д., 24 роки, потрапила до лікарні зі скаргами

на біль голови, біль в поперековій ділянці, набряки на

обличчі, загальну слабість. Місяць тому перенесла

ангіну. При поступленні: АТ-180/110 мм рт. ст. У сечі:

виражена протеїнурія, мікрогематурія, лейкоцитурія.

Якою формою гіпертензії страждає хвора?

A. *Нирковою

B. Гіпертонічною хворобою

C. Есенціальною

D. Первинною

E. Ендокринною

26. Хворому Н. 3 роки тому було встановлено діагноз

хронічний гломерулонефрит. Протягом останніх 6

місяців з’явились набряки. Що лежить в основіїх

розвитку?

A. *Протеїнурія

B. Гіперпродукція вазопресину

C. Порушення білковоутворюючої функції печінки

D. Гіперосмолярність плазми

E. Гіперальдостеронізм

27. У хворого з первинним нефротичним синдромом

вміст загального білка крові 40 г/л. Яка причина зумо-

вила гіпопротеїнемію?

A. *Протеїнурія

B. Вихід білка з судин у тканини

C. Зниження синтезу білка у печінці

D. Підвищений протеоліз

E. Порушення всмоктування білка у кишечнику

28. У хворого на хронічний гломерулонефрит швид-

кість клубочкової фільтрації (ШКФ) знижена до 20% від

нормальної. Що спричинює зниження ШКФ при хроніч-

ній нирковій недостатності?

A. *Зменшення кількості діючих нефронів

B. Тубулопатія

C. Обтурація сечовивідних шляхів

D. Ішемія нирок

E. Тромбоз ниркових артерій

29. У хворого, 58 років, з гострою серцевою недостані-

стю, спостерігалось зменшення добової кількості сечі

– олігоурія. Який механізм цього явища?

A. *Зниження клубочкової фільтрації.

B. Зниження кількості функціонуючих клубочків.

C. Зниження онкотичного тиску крові.

D. Підвищення гідростатичного тиску на стінку капіля-

рів.

E. Зниження проникності клубочкової мембрани.

30. У хворого, віком 30 років, який потрапив до клініки

з діагнозом “гострий гломерулонефрит”, спостеріга-

лась протеїнурія. Яке порушення спричинило це яви-

ще?

A. *Підвищення проникності клубочкового фільтру.

B. Затримка виведення продуктів азотистого обміну.

C. Зниження онкотичного тиску крові.

D. Підвищення гідростатичного тиску на стінку капіля-

рів.

E. Зниження кількості функціонуючих нефронів.

31. У результаті землетрусу чоловік 50 років два дні

перебував під завалом. Яке ускладнення найбільш

вірогідне після звільнення з-під завалу?

A. *Гостра ниркова недостатність.

B. Гостра печінкова недостатність.

C. Гостра серцева недостатність.

D. Гостра судинна недостатність.

E. Гостра дихальна недостатність.

32. У дитини 5 років через 2 тижні після перенесеної

ангіни виник гострий дифузний гломерулонефрит, що

характеризувався олігурією, протеїнурією, гематурією,

гіперазотемією. Порушення якої функції нефронів

найбільш суттєве для виникнення цих симптомів?

A. *Клубочкової фільтрації.

B. Канальцевої реабсорбції.

C. Канальцевої секреції.

D. Сечовиведення.

E. Інкреторної функції.

33. В експерименті кролю ввели нефроцитотоксичну

сироватку морської свинки. Яке захворювання нирок

моделювалося в цьому досліді?

A. * Гострий дифузний гломерулонефрит

B. Нефротичний синдром

C. Гострий пієлонефрит

D. Хронічна ниркова недостатність

E. Хронічний пієлонефрит

34. Після масивної крововтрати у хворого відзначаєть-

ся олігоурія, гіперазотемія, набряк мозку та легень. У

зниженні діурезу в даному випадку має значення:

A. *Зниження ефективного тиску фільтрації

B. Збільшення реабсорбції води в канальцях

C. Збільшення реабсорбції натрію в канальцях

D. Перерозподіл води в організмі

E. Зниження проникності клубочкової мембрани

35. В результаті порушення техніки безпеки відбулося

отруєння сулемою. Через 2 дні добовий діурез склав

620 мл. У хворого з’явилися головний біль, блювота,

судоми, задишка, в легенях – вологі хрипи. Який

патологічний стан виник у хворого?

A. *Гостра ниркова недостатність

B. Хронічна ниркова недостатність

C. Уремічна кома

D. Гломерулонефрит

E. Пієлонефрит

36. У хворого А., 38 років, на 3-му році захворювання

системним червоним вовчаком виникло дифузне

ушкодження нирок, що супроводжується масивними

набряками, вираженою протеїнурією, гіперліпідемією,

диспротеїнемією. Який найбільш вірогідний механізм

розвитку протеїнурії в даному випадку?

A. *Аутоімунне пошкодження нефронів

B. Запальне пошкодження нефронів

C. Ішемічне пошкодження канальців

D. Збільшення рівня протеїнів в крові

E. Ушкодження сечовивідних шляхів

37. Хворий А., 27 років, госпіталізований з шлунковою

кровотечею у важкому стані. АТ - 80/60 мм.рт.ст. До-

бовий діурез 60 - 80 мл, питома вага 1,028-1,036. Який

патогенетичний механізм зумовив олігоурію?

A. *Зниження гідростатичного тиску в капілярах клубо-

чків

B. Підвищення осмотичного тиску сечі

C. Високий рівень залишкового азоту в крові

D. Підвищення колоїдно-осмотичного тиску в крові

E. Підвищення гідростатичного тиску в капсулі Шум-

лянського-Боумена

38. Хворому поставлене діагноз: ниркова артеріальна

гіпертензія. Назвіть ініціюючий патогенетичний чинник

розвитку артеріальної гіпертензії в даному випадку?

A. * Ішемія нирок

B. Гіпернатріємія

C. Гіперальдостеронізм

D. Збільшення синтезу реніну

E. Збільшення синтезу ангіотензину

39. У хворого з цукровим діабетом розвинулась неф-

ропатія, ускладнена уремією. Швидкість клубочкової

фільтрації 9 мл/хв. Який найбільш вірогідний механізм

зниження швидкості клубочкової фільтрації і розвитку

ХНН у пацієнта?

A. * Зменшення маси діючих нефронів

B. Зниження системного артеріального тиску

C. Закупорка просвіту канальців нефрону гіаліновими

циліндрами

D. Розвиток ацидозу в тканинах

E. Спазм приносних артеріол

40. У добовій сечі хворого виявлено вилужені еритро-

цити. Для якої патології нирок найбільш характерний

виявлений симптом?

A. * Дифузний гломерулонефрит

B. Нефротичний синдром

C. Нирково-кам’яна хвороба

D. Пі .лонефрит

E. Гостра ниркова недостатність

41. Які зміни в сечі свідчать про підвищення проникно-

сті клубочкової мембрани?

A. * Протеїнурія

B. Глюкозурія

C. Аміноацидурія

D. Алкаптонурія

E. Піурія

42. Чим викликані оліго- та анурія в термінальній стадії

ХНН?

A. * Зменьшенням кількості діючих нефронів

B. Ішемією коркової речовини нирок внаслідок спазму

судин

C. Зменшенням фільтраційного тиску і фільтрації

D. Збільшенням реабсорбції води в дистальних ка-

нальцях

E. Дисемінованим внутрішньосудинним зсіданням

крові

43. У хворого з хронічним пієлонефритом спостеріга-

ється добовий діурез 4 л, відносна щільность сечі до

1,010. Яка з функцій нирок порушена?

A. *Реабсорбція натрію і води

B. Клубочкова фільтрація

C. Канальцева секреція216

D. Реабсорція глюкози

E. Реабсорбція білка

44. У хворого з гострою нирковою недостатністю на 6-

й день проведення терапевтичних заходів виникла

поліурія. Зростання діурезу на початку поліуричної

стадії гострої ниркової недостатності зумовлене:

A. *Відновленням фільтрації в нефронах

B. Збільшенням об’єму циркулюючої крові

C. Збільшенням натрійуретичного чинника

D. Зменшенням альдостерону в плазмі

E. Зменшенням вазопресину в плазмі

45. У жінки 30 років виникли набряки обличчя. При

обстеженні виявлено білок в сечі 5,87 г/л, білок в крові

50 г/л, диспротеїнемію, гіперліпідемію. Поєднання цих

ознак характерне для?

A. *Нефротичного синдрому

B. Нефритичного синдрому

C. Хронічного пієлонефриту

D. Гострої ниркової недостатності

E. Хронічної ниркової недостатності

46. Хворий знаходився під завалом 4 годин. Госпіталі-

зований з ознаками набряку головного мозку, діурезу

200 мл/добу, гіперазотемії. У якій стадії гострої нирко-

вої недостатності знаходиться хворий?

A. * Олігоануричній

B. Початковій

C. Поліурічній

D. Одужання

E. -

47. Хвора Н., 55 років, хворіє на хронічний гломеруло-

нефрит впродовж 15 років. Які зміни свідчать про

обмеження секреторної функції нирок?

A. * Гіперазотемія

B. Гіперглікемія

C. Гіпопротеїнемія

D. Протеїнурія

E. Гіпоізостенурія

48. У хворого після важкої травми грудної клітки роз-

винувся шок та з’явилися ознаки гострої ниркової

недостатності (ГНН). Що є провідним механізмом

розвитку ГНН в даному випадку?

A. * Падіння артеріального тиску

B. Порушення відтоку сечі

C. Підвищення тиску в капсулі клубочка

D. Підвищення тиску в ниркових артеріях

E. Зменшення онкотичного тиску крові

49. У чоловіка 25 років діагностовано гострий дифуз-

ний гломерулонефрит. З анамнезу відомо, що за 18

днів тому було перенесено ангіну. Який механізм

пошкодження ниркових клубочків буде спостерігатися

у цьому випадку?

A. *Імунний.

B. Ацидотичний.

C. Нефротоксичний.

D. Ішемічний.

E. Медикаментозний.

50.. З порушенням якої функції нирок пов’язано змен-

шення кліренсу за інуліном до 60 мл/хв?

A. * Клубочкової фільтрації

B. Канальцевої секреції

C. Реабсорбції в проксимальному відділі нефрону

D. Реабсорбції в дистальному відділі нефрону

E. Реабсорбції в збиральних ниркових трубочках

51. У хворого на хронічну ниркову недостатність

з’явилися анорексія, диспепсія, порушення ритму

серця, свербіння шкіри. Який механізм розвитку цих

порушень є головним?

A. * Накопичення продуктів азотистого обміну в крові

B. Порушення ліпідного обміну

C. Зміни вуглеводного обміну

D. Нирковий ацидоз

E. Порушення водно-електролітного обміну

52. У хворого з гломерулонефритом виявлено: набря-

ки по всьому тілу, АТ 185/105 мм рт.ст. У крові:

2,6*1012г/л, лейк. - 9,9*109г/л, гіперазотемія, білок 42

г/л. Який показник свідчить про ускладнення гломеру-

лонефриту нефротичним синдромом?

A. *Гіпопротеїнемія

B. Лейкоцитоз

C. Гіперазотемія

D. Артеріальна гіпертензія

E. Анемія

53. 43-річний хворий поступив в нефрологічне відді-

лення з масивними набряками. Два роки лікувався

амбулаторно і при цьому постійно відзначався підви-

щений АТ. Двічі лікувався преднізолоном, з позитив-

ним ефектом. У сечі: відносна щільність 1017, білок

4,0 г/л, Ер – 15-18 в полі зору (вилужені), Лей - 5-7 в

полі зору. Яка функція нирок найвірогідніше порушена

у хворого?

A. *Фільтраційна

B. Реабсорбційна

C. Секреторна

D. Інкреторна

E. Концентраційна

54. У кролика відтворили хронічний гломерулонефрит

шляхом введення великих доз протиниркової сироват-

ки морської свинки. Що лежить в основі його розвитку?

A. *Алергічний процес

B. Некроз епітелію канальців

C. Гломерулосклероз

D. Амілоїдоз нирок

E. Азотемія

55. У результаті гострої ниркової недостатності у

хворого виникла олігурія. Яка добова кількість сечі

відповідає даному симптому?

A. *100-500 мл

B. 1500-2000 мл

C. 1000-1500 мл

D. 500-1000 мл

E. 50-100 мл

56. У хворого з патологією нирок виявлено масивну

протеїнурію, набряки, гіпопротеїнемію, ретенційну

гіперліпідемію. Як називається цей патологічний про-

цес?

A. *Нефротичний синдром

B. Гіпертензивний синдром

C. Сечовий синдром

D. Анемічний синдром

57. У хворого з хронічною патологією нирок після

проведення проби Зимницького виявлено ізогіпосте-

нурію. Які зміни сечовиділення будуть спостерігатися

при цьому?

A. *Поліурія

B. Олігурія

C. Анурія

D. Полакіурія

E. Ніктурія

58. У хворого на мієломну хворобу виявили білок в

сечі. Яка форма протеїнурії має місце у даного хворо-

го?

A. * Супраренальна.

B. Ренальна гломерулярна.

C. Ренальна тубулярна.

D. Субренальна набрякова.

E. Субренальна уретральна.

59. Кролю внутрішньовенно ввели нефротоксичну

сироватку гвінейської свинки, яка була попередньо

імунізована суспензією нирки кроля. Яка патологія

нирок моделюється таким чином?

A. *Гломерулонефрит

B. Пієлонефрит

C. Ниркова недостатність

D. Нефротичний синдром

E. Тубулярна недостатність

60. У хворого з ГНН у стадії поліурії азотемія не тільки

не зменшилася, але продовжувала наростати. З по-

рушенням якої функції нирок це пов’язано?

A. *Секреція

B. Фільтрація

C. Реабсорбція

D. Інкреція

E.

61. У піддослідної тварини в експерименті видалено

одну нирку та накладено звужуючу лігатуру на артерію

іншої нирки. Який вид вторинної гіпертензії виник у

неї?

A. *Реноваскулярна

B. Ренопривна

C. Ендокринна

D. Нейрогенна

E. Ангіогенна

62. У хворої з феохромоцитомою після психічного

навантаження виникає тахікардія, підвищується арте-

ріальний тиск, з’являється різкий біль у надчеревній

ділянці. Ці приступи зумовлені:

A. *Масивним викидом катехоламінів наднирниками

B. Звільненням норадреналіну симпатичними нервами

C. Активацією вегетативних ядер гіпоталамуса

D. Збільшенням секреції тиреоїдних гормонів

E. Підвищеним синтезом адренокортикотропного

гормону

63. Чоловік 53 років, який страждає на подагру, скар-

житься на біль у попереку. При ультразвуковому

обстеженні виявлено ниркові камені. Підвищення

вмісту якої речовини є ймовірною причиною утворення

каменів?

A * Сечової кислоти

B Цистину

C Білірубіну

D Сечовини

E Холестерину

64. Хворий тривалий час страждає на гіпертонічну

хворобу. Який аналіз крові

потрібно час від часу робити, щоб вчасно виявити

початок розвитку ниркового

ускладнення?

A *Визначення вмісту креатиніну.

B Визначення вмісту білка.

C Визначення вмісту натрію.

D Визначення вмісту калію.

E Визначення вмісту холестерину.

65. Чоловік 35 років страждає на хронічний гломеру-

лонефрит. Обстеження у нефрологічному відділенні

виявило ізольований сечовий синдром у вигляді

помірної протеїнурії і гематурії. Яка клінічна форма

хронічного гломерулонефриту має місце у хворого?

A *Латентна форма.

B Гіпертензивна форма.

C Нефротична форма.

D Нефротично-гіпертензивна форма.

E .

66. В нефрологічному відділі у хворого з

пієлонефритом при обстеженні було виявлено

гіпостенурію в поєднанні з поліурією. Який процес

порушений?

A *Канальцева реабсорбція

B Клубочкова фільтрація

C Канальцева секреція

D Канальцева екскреція

E -

67. В стаціонар поступив хворий з підозрою на ГНН.

Які зміни показників крові матимуть місце?

A *Підвищений вміст сечовини

B Знижений вміст сечової кислоти

C Знижений вміст кальцію

D Знижений вміст калію

E Знижений вміст фосфору

68. Після перенесеної стрептококової інфекції у чоло-

віка діагностовано гострий гломерулонефрит.

Найбільш вірогідно, що ураження базальної мембрани

клубочків являє собою алергічну реакцію:

A *Імунокомплексного типу

B Анафілактичного типу

C Цитотоксичного типу

D Сповільненого типу

E Стимулюючого типу

69. На фоні захворювання печінки та нирок в крові

зростає вміст залишкового азоту. За рахунок якої

фракції зростає вміст залишкового азоту в хворих з

патологією нирок?

A * Сечовини

B Сечової кислоти

C Аміаку

D Амонія

E Прокінази

70. У водія, який потрапив у ДТП, спостерігається

зменшення добової кількості сечі до 300 мл. Який

основний патогенетичний фактор цієї зміни діурезу?

A Падіння артеріального тиску

B Зниження онкотичного тиску крові

C Підвищення проникності судин

D Зменшення кількості функціонуючих клубочків

E Вторинний гіперальдостеронізм

71. На фоні деяких отруєнь глюкоза з’являється в сечі

при збереженні нормального рівня в крові. Який відділ

нефронів уражає токсична речовина?

A * Проксимальні канальці

B Клубочки

C Петля Генле

D Дистальні канальці

E Збірні трубочки

72. У хворого на хронічний тонзилит з’явились набряки

під очима, підвищена втомлюваність. Артеріальтний

тиск 170/110 мм рт.ст. У сечі виявлено білок – 0,99%.

Про яку патологію слід думати?

A * Нефрит

B Пієліт

C Цистит

D Уретрит

E Гепатит__217

73. У чоловіка віком 57 років, який страждав хронічним

пієлонефритом, була виявлена артеріальна

гіпертензія. Який механізм підвищення артеріального

тиску є головним у цьому випадку?

A *Збільшення секреції реніну нирками.

B Збудження вегетативних центрів гіпоталамуса.

C Підвищення рівня катехоламінів в крові.

D Збудження барорецепторів синокаротидної зони.

E Збудження кори великих півкуль.

74. Хвора 58 років скаржиться на періодичний біль в

поперековій області, що супроводжується лихоман-

кою, слабістю, помірною поліурією та набряками. В

сечі: лейкоцити 30-40 в колі зору, еритроцити 10-15 в

колі зору, білок 0,4 г/л, бактерійурія. Яка патологія

нирок найбільш вірогідна у хворої?

A *Пієлонефрит.

B Гострий гломерулонефрит.

C Хронічний гломерулонефрит.

D Нефротичний синдром.

E Нефролітіаз.

75. При захворюваннях нирок, які супроводжуються

порушенням екскреторної функції клубочків (гострий і

хронічний гломерулонефрит) часто спостерігається

анемія, що носить нормоцитарний, нормохромний та

гіпорегенеративний характер. Яка найбільш вірогідна

причина анемії?

A *Підвищення секреції інгібітора еритропоезу.

B Дефіцит заліза.

C Пригнічення функцій кісткового мозку

азотвмісними речовинами.

D Гематурія.

E Геморагічний діатез.

76. Внаслідок масивної крововтрати у хворого споспо-

стерігається зниження АТ 80/50 мм рт.ст., збільшені

ЧСС та дихання. Лікар запідозрив розвиток постгемо-

ррагічного шоку. Яка функція нирок порушена в першу

чергу?

A *Фільтраційна.

B Секреторна.

C Концептраційна.

D Еритропоетична.

E Азотовидільна.

77. У хворого при обстеженні виявлена глюкозурія,

гіперглікемія. Скарги на сухість в роті, свербіння шкіри,

часте сечовиділення, спрагу. Поставлен діагноз: цук-

ровий діабет. Чим зумовлена поліурія у даного хворо-

го?

A *Збільшенням осмотичного тиску сечі.

B Зменьшенням онкотичного тиску плазми.

C Збільшенням фільтраційного тиску.

D Зменьшенням серцевого викиду.

E Збільшенням онкотичного тиску плазми.

78. У хворого з масивними опіками в першу добу

розвинулась гостра недостатність нирок, що

характеризується значним зменшенням швидкості

клубочкової фільтрації. Який механізм розвитку пато-

логічного процесу?

A *Зменшення ниркового кровоплину

B Ушкодження клубочкового фільтра

C Зменшення кількості функціонуючих нефронів

D Збільшення тиску канальцевої рідини

E Емболія ниркової артерії

79. Внаслідок множинних переломів кісток і розвитку

шоку в постраждалого припинилось виділення сечі, що

може бути зумовлено падінням систолічного

артеріального тиску нижче (мм рт.ст.)

A * 60

B 65

C 70

D 75

E 80

80. Одним із симптомів інсулінової недостатності є

поліурія. Що являється причиною цього?

A * Осмотичний діурез по причині гіперглікемії

B Гіперпродукція вазопресину

C Гіпоглікемія

D Недостатнє вироблення атріопептиду

E Порушення реабсорбції натрію

81. В експерименті тварині зменшили кровопостачан-

ня нирок. Через деякий час різко підвищився кровяний

тиск. Що було причиною цього явища?

A *Підвищення продукції реніну

B Зменшення клубочкової фільтрації

C Порушення процесів реабсорбції в проксимальних

звивистих канальцях

D Порушення процесів реабсорбції в дистальних

звивистих канальцях

E Зменшення ефективного фільтраційноготиску

82. У пацієнта генетично сформована недостатність

транспортних систем для реабсорбції глюкози в про-

ксимальних звивистих канльцях. Який симптомоком-

плекс буде при цьому?

A *Глюкозурія при нормальному вмісту глюкози в крові

B Поліурія

C Полідіпсія

D Натрійурія

E Зменшення продукції інсуліну

83. Внаслідок землетрусу чоловік 50-ти років двi доби

перебував під завалом. Після звільнення з-під завалу

рятівниками у нього був встановлений синдром трива-

лого розчавлення. Виникнення якого ускладнення в

подальшому найбільш вірогідне?

A. *Гостра ниркова недостатність

B. Гостра дихальна недостатність

C. Гостра судинна недостатність

D. Гостра печінкова недостатність

E. Гостра серцева недостатність

84. У хворого 38-ми років на 3-му році захворювання

на системний червоний вівчак виявлене дифузне

ураження нирок, що супроводжується масивними

набряками і вираженою протеїнурією. Що є найбільш

вірогідною причиною розвитку протеїнурії у пацієнта?

A. *Аутоімунне ушкодження нирок

B. Ішемічне ушкодження нирок

C. Асептичне ураження нирок

D. Запальне ураження сечовивідних шляхів

E. Запальне ураження сечового міхура

85. У хворого, який скаржиться на поліурію і

полідипсію, знайдено цукор в сечі. Вміст цукру в

плазмі крові у нормі. З чим пов'язаний механізм

глюкозурії у хворого?

A. *Порушення реабсорбції глюкози в канальцях неф-

рону

B. Порушення фільтрації глюкози в клубочковому

відділі нефрону

C. Гіперпродукція глюкокортикоїдів наднирниками

D. Недостатня продукція інсуліну підшлунковою зало-

зою

E. Інсулінорезистентність рецепторів клітин

Патофізіологія ендокринної системи

1. У хворого Н., 50 років, зі скаргами на втрату маси

тіла, слабість, в крові виявлено гіпоглікемію і гіперін-

сулінемію. При додатковому обстеженні виявлено

пухлину острівців Лангерганса. Посилений синтез

інсуліну в даному випадку розцінюється як:

A. *Функціональний

B. Морфологічний

C. Біохімічний

D. Фізико-хімічний

E. Імунологічний

2. У юнака 20 років травмоване праве яєчко. Яку не-

безпеку це може становити для лівого (здорового)

яєчка на 2-3 тижні опісля?

A. *Демаскування антигену та виникнення ушкодження

антитілами

B. Розвиток інфекційного процесу

C. Розвиток атрофії

D. Розвиток гіпертрофії

E. Не загрожує нічим

3. У дитини 6 років розвинулася гіперергічна форма

запалення верхніх дихальних шляхів. З’явилася загор-

за серйозного порушення дихання, а тому виникла

необхідність застосувати протизапальні гормони.

Серед гормонів протизапальний ефект проявляє

A. * Кортизол

B. Адреналін

C. Соматотропін

D. Тестостерон

E. Інсулін

4. До лікаря звернувся чоловік 27 років. При огляді

було виявлено збільшення китиць, стоп та нижньої

щелепи. Крім того спостерігалась деформація сугло-

бів, гормональні порушення (імпотенція, атрофія

яєчок). Функції якої залози порушені?

A. * Передньої частини гіпофізу

B. Надниркових залоз

C. Шишкоподібного тіла

D. Щитоподібної залози

E. Прищитоподібних залоз

5. Хворий скаржиться на підвищену дратівливість,

періодичний субфібрилітет. Частота пульсу - 120 за

хв. У крові збільшена кількість гормонів Т3 і Т4. Яку

ендокринну патологію найбільш логічно запідозрити?

A. * Гіпертиреоз.

B. Надниркову недостатність.

C. Гіпопаратиреоз.

D. Гіперпаратиреоз.

E. Гіпотиреоз.

6. У хворої А., 18 років, після перенесеної краснухи

почала відзначатись втрата маси тіла, постійне відчут-

тя сухості в роті, спрага, підвищення апетиту, часте

сечовиділення. Об’єктивно: добова кількість сечі 6 л,

глюкоза крові 17,8 ммоль/л, в сечі виявлена глюкоза і

ацетон. Який найбільш вірогідний патогенетичний

механізм викликав підвищення рівня глюкози у хворої?

A. *Зменшення вироблення інсуліну

B. Збільшення глюконеогенезу

C. Підвищене руйнування інсуліну

D. Пошкодження інсулінових рецепторів клітин

E. Збільшення вироблення глюкокортикоїдів

7. Хворий з Прикарпаття, що страждає на ендемічний

зоб, звернувся до лікаря із скаргами на гноєтечі з

ясеневих закутків та розхитування зубів. Що в даному

випадку є основним чинником розвитку пародонтиту?

A. * Ендокринні порушення

B. Стресові впливи

C. Гіперсалівація

D. Порушення ковтання

E. Неповноцінне харчування

9. У хворого спостерігаються напади артеріальної

гіпертензії, що супроводжуються тахікардією, рясним

потовиділенням, різким болем в надчеревній ділянці.

Для якого з перелічених пухлинних захворювань най-

більш характерні дані симптоми?

A. *Феохромоцитома

B. Базофільна аденома гіпофіза

C. Аденома клубочкової зони наднирників

D. Аденома щитоподібної залози

E. Пухлина яєчників

10. У піддослідної тварини (щура) шляхом внутріш-

ньовенного введення алоксану був викликаний експе-

риментальний цукровий діабет. Який механізм дії

даної речовини?

A. * Пошкодження beta - клітин панкреатичних острів-

ців

B. Скріплення цинку

C. Утворення антитіл до інсуліну

D. Активація інсулінази

E. Активація вироблення контрінсулярних гормонів

11. Жінка 53 років, ріст 163 см, вага тіла 92 кг, відміча-

ється: рівномірне відкладання жиру, одутловатість

обличчя, сонливість, апатичність. При натискуванні на

шкіру гомілки залишається ямка. Порушенням функції

якої залози зумовлений стан хворої?

A. *Щитоподібної

B. Гіпофізу

C. Надниркових

D. Статевих.

E. Прищитовидних.

12. У жінки 46 років після операції на щитовидній

залозі невдовзі з’явилися фібрилярні посмикування

м’язів рук, ніг, обличчя. Ці порушення можна усунути

шляхом введення:

A. *Паратгормону

B. Трийодтироніну

C. Тиреотропіну

D. Тироксину

E. Тиреотропного гормону

13. У хворого 48 років спостерігається артеріальна

гіпертензія, головний біль, м’язoва слабість, судоми. У

крові знижена концентрація К+ і підвищена концентра-

ція Na+, що є наслідком гіперсекреції:

A. *Альдостерону

B. Адреналіну

C. Паратгормону

D. Кортизолу

E. Дигідрохолестеролу

14. Жінка 38 років скаржиться на загальну слабість,

біль в ділянці серця, підвищення апетиту, відсутність

менструацій. Об’єктивно: зріст 166 см, вага тіла 108 кг,

обличчя місяцеподібне, відкладення підшкірної клітко-

вини переважно в ділянці верхнього плечового поясу,

тулубу; на шкірі стегон, живота багряно-червоні смуги,

пульс 62 уд. за хв., АТ-160/105 мм. рт. ст. Для якого із

перелічених нижче станів це найбільш характерно?

A. *Хвороба Іценко-Кушинга

B. Аліментарне ожиріння

C. Мікседема

D. Інсулінома__218

E. Синдром Бабінського-Фреліха

15. Жінка 26 років, скарги на наявну загальну слабість,

втрату маси тіла на 18 кг, відсутність менструацій,

хворіє вже 1 рік, після пологів; пологи були важкі,

супроводжувались кровотечею. Об’єктивно: зріст 168

см, вага тіла 53 кг гіпоплазія молочних залоз. Діагнос-

товано синдром Шихана. Що є основним механізмом

втрати ваги у жінки?

A. *Зниження продукції гормонів аденогіпофізу

B. Зниження функції статевих залоз

C. Зниження функції кіркового кулі наднирників

D. Гіпотиреоз

E. Гіпопаратиреоз

16. Після двостороннього видалення наднирників у

собаки з’явилася м’язова слабість, адинамія, знижен-

ня температури тіла, гіпоглікемія. Які ще можуть бути

зазначені прояви надниркової недостатності?

A. *Артеріальна гіпотонія

B. Лімфопенія

C. Посилення синтезу глікогену

D. Збільшення концентрації натрію і хлоридів в сиро-

ватці крові

E. Підвищення резистентності до дії бактерій і токсинів

17. У хворих з тиреотоксикозом підвищення темпера-

тури тіла виникає у зв’язку з:

A. *Підвищенням активності біологічного окислення,

роз’єднанням процесів окислення і фосфорилювання

B. Збільшенням поглинання кисню організмом

C. Посиленням теплоутворення в печінці

D. Звуженням периферичних судин

E. Активацією катаболізму білків

18. Хворий А., 18 років, після перенесеної краснухи

почавши худнути, постійно відчував сухість в роті,

спрагу, у нього підвищився апетит, почалося часте

сечовиділення. Об’єктивно: добова кількість сечі 6 л,

глюкоза крові 17 ммоль/л, в сечі виявлена глюкоза та

ацетон. Яку захворювання виникло у хворого?

A. *ІЗЦД (інсулінзалежний цукровий діабет).

B. Симптоматичний цукровий діабет.

C. Вторинний цукровий діабет.

D. ІНЦД (інсуліннезалежний цукровий діабет).

E. Стероїдний діабет.

19. У жінки 55 років, яка проживає в гірській місцевості,

діагностований ендемічний зоб. Об’єктивно: надмірна

вага тіла, загальмована, апатична, збільшення щито-

видної залози. Дефіцит якого з перелічених нижче

елементів викликає захворювання?

A. *Йод

B. Фтор

C. Марганець

D. Молібден

E. Натрій

20. У хлопчика діагностовано ендемічний зоб. Який

основний механізм розвитку гіпотиреозу у хлопця?

A. *Зниження продукції тироксину та трийодтироніну

B. Зниження продукції тиреотропіну

C. Зниження чутливості рецепторів тканин до тирокси-

ну та трийодтироніну

D. Підвищення метаболізму тироксину та

трийодтироніну

E. Порушення транспорту тироксину та

трийодтироніну

21. Чоловіку 46 років, який хворіє дифузним токсичним

зобом, була проведена операція резекції щитоподібної

залози. Після операції відмічаються відсутність апети-

ту, диспепсія, підвищена нервно-м’язова збудливість.

Маса тіла не збільшилась. Температура тіла нормаль-

на. Чим із нижче переліченого зумовлений стан чоло-

віка?

A. *Зниженням продукції паратгормону

B. Зиженням продукції тироксину

C. Підвищенням продукції кальцитонину

D. Підвищенням продукції тиреоліберіну

E. Порушення продукції тироксину

22. Чоловік 42 років, що страждає ожирінням за верх-

нім типом (верхній плечовий пояс, обличчя місяцепо-

дібне), АТ - 160/95 мм рт.ст., глюкоза крові 8,0

ммоль/л. Вміст кортизолу в крові підвищений, а адре-

нокортикотропін знижений. Яка найбільш вірогідна

причина розвитку гіперкортицизму?

A. *Гормонопродукуюча пухлина кори наднирників

B. Гормонопродукуюча пухлина передньої ділянки

гіпофізу

C. Зменшення продукції статинів

D. Збільшення продукції кортиколіберину

E. Зменшення вироблення статевих гормонів

23. Методом непрямої калориметрії встановлено, що

основний обмін досліджуваного на 40 % нижче належ-

ного. Порушення діяльності якої ендокринної залози

можна припустити?

A. *Щитоподібної залози.

B. Тимусу.

C. Підшлункової залози.

D. Епіфіза

E. Прищитоподібних залоз

24. У 50-річної хворої після перенесеного інфекційного

захворювання головного мозку значно збільшився

діурез до 12 л за добу. При аналізі крові кількість

глюкози становила 4,1 ммоль/л. Якого гормону найві-

рогідніше недостатньо?

A. *Антидіуретичного

B. Глюкагону

C. Інсуліну

D. Кортизону

E. Альдостерону

25. У клініці встановлено, що при вагітності тяжкість

симптомів ревматоїдного артриту різко знижується.

Прискорення секреції яких гормонів, що володіють

протизапальною дією, спостерігається при цьому?

A. *Глюкокортикоїдів

B. Естрогену

C. Йодидів щитоподібної залози

D. Катехоламінів

E. Гонадотропних

26. У хворого має місце хронічна недостатність кірко-

вої речовини наднирникових залоз (Аддісонова або

бронзова хвороба). Недостатність якого гормону має

місце при цьому патологічному процесі?

A. *Альдостерону

B. Інсуліну

C. Адреналіну

D. Тироксину

E. Вазопресину

27. Через 1 – 2 доби після видалення у собаки прищи-

топодібних залоз спостерігались: млявість, спрага,

різке підвищення нервово-м’язовоїзбудливостіз

розвитком тетані .. Яке порушення обміну електролітів

має місце?

A. *Гіпокальціємія

B. Гіперкальціємія

C. Гіпомагніємія

D. Гіпермагніємія

E. Гіпонатріємія

28. Жінка 29 років скаржиться на загальну слабість,

втрату маси тіла на 22 кг, аменорею. Хворіє після

пологів. Об’єктивно: ріст 162 см, маса 46 кг, гіпоплазія

молочних залоз. Діагностована гіпофізарна кахексія.

Зменшення продукції якого гормону з’явилося най-

більш суттєвим у схудненні жінки?

A. *Соматотропіну.

B. Адренокортикотропіну.

C. Тиреотропіну.

D. Меланотропіну.

E. Пролактотропіну.

29. У хворої після пологів развинулись такі ознаки:

атрофія скелетних м’язів, дистрофія шкіри, випадання

волосся, гіпотрофія внутрішніх органів, зниження

температури тіла, артеріального тиску, рівня глюкози в

крові, атрофія щитоподібної, наднирникових та стате-

вих залоз. Для якої патології характерні ці ознаки?

A. *Атрофії гіпофізу

B. Пошкодження статевих залоз

C. Гіпофункції щитоподібної залози

D. Гіперфункції аденогіпофізу

E. Пошкодження наднирникових залоз

30. У ліквідатора аварії на Чорнобильській АЕС через

деякий час з’явилися скарги на підвищену збудливість,

нервозність, серцебиття, зниження маси тіла, постійну

слабкість, тремтіння тіла, відчуття жару, погану пере-

носимість тепла. Гіперфункція якої залози може бути

причиною вказаних змін:

A. *Щитоподібної залози

B. Аденогіпофізу

C. Кіркової речовини надниркових

D. Мозкової речовини надниркових

E. Паращитоподібної залози

31. Хворому 45-ти років при оперативному втручанні

на щитоподібній залозі випадково видалили прищито-

подібні залози. Це призвело до:

A. * Тетанії

B. Підвищення рівня кальцію в крові і резорбції кісток

C. Підвищення рівня кальцію, натрію і калію в крові

D. Зниження артеріального тиску

E. Підвищення артеріального тиску

32. У пацієнта, носія спадкової серповидної аномалії

еритроцитів, захворювання пневмонією супроводжу-

валося гемолітичним кризом і розвитком анемії. Що є

безпосередньо причиною гемолітичного кризу в дано-

му випадку?

A. *Гіпоксія, викликана пневмонією

B. Зміна осмолярності крові

C. Гетерозиготність по Нb S

D. Мутація структурного гена

E. Гіпероксія

33. У собаки з ендокринною патологією було виявле-

но: зменшення споживання кисню у стані спокою,

зниження толерантності до глюкози, зниження темпе-

ратури тіла. Недостатністю якого гормону можна

пояснити виявлені зміни?

A. *Тироксину

B. Інсуліну

C. Соматотропіну

D. Адренокортикотропіну

E. Гонадотропіну

34. Жінка 44 років скаржиться на загальну слабість,

біль в ділянці серця, значне збільшення маси тіла.

Об’єктивно: обличчя місяцеподібне, гірсутизм, АТ -

165/100 мм рт.ст., зріст - 164 см, вага - 103 кг, перева-

жно накопичення жиру на шиї, верхньому плечовому

поясі, животі. Що є основним патогенетичним механі-

змом ожиріння у жінки?

A. *Підвищення продукції глюкокортикоїдів

B. Зниження продукції тиреоїдних гормонів

C. Підвищення продукції інсуліну

D. Зниження продукції глюкагону

E. Підвищення продукції мінералокортикоїдів

35. Після інсульту з ураженням ядер гіпоталамусу у

хворого виник нецукровий діабет. Що стало причиною

посиленого сечовиділення у цього хворого?

A. *Зменшення реабсорбції води

B. Зменшення реабсорбції натрію

C. Зниження артеріального тиску

D. Гіперглікемія

E. Прискорення клубочкової фільтрації

36. У хворої віком 39 років були видалені яєчники з

приводу злоякісної пухлини. Через 2 роки у неї

з’явились гірсутизм, огрубіння голосу, статура набула

чоловічих ознак. Яке із гормональних зрушень лежить

в основі даного стану?

A. *Зниження естрогенів

B. Відсутність прогестерону

C. Посилення продукції пролактину

D. Зниження продукції андрогенів

E. Гіпофункція наднирникових залоз

37. У хворого на цироз печінки з’явилась артеріальна

гіпертензія, м’язова слабість, періодичні судоми. У

крові – збільшений вміст Nа і зменшений вміст К. Який

із видів ендокринних порушень лежить в основі даного

симптомокомплексу?

A. * Вторинний альдостеронізм.

B. Гіпопітуітаризм.

C. Первинний альдостеронізм.

D. Гіперпітуітаризм.

E. Гіпоальдостеронізм.

38. Хвора, 28 р., скаржиться на в’ялість, швидку розу-

мову та фізичну втомлюваність, диспептичні порушен-

ня. При обстеженні виявлено: позитивні туберкулінові

проби, гіпоглікемія, АТ – 90/60 мм.рт.ст., гіпонатріємію,

пігментацію шкіри. При якій патології наднирників

спостерігаються подібні явища?

A. * Хвороба Аддісона.

B. Синдром Іценка-Кушинга.

C. Гостра недостатність кори наднирників.

D. Гіпофункція мозкового шару наднирників.

E. Синдром Конна.

39. У хворого, що тривалий час лікувався з приводу

однієї із форм колагенозу препаратами кортикостерої-

дних гормонів, з’явилися швидка фізична і психічна

втомлюваність, схуднення, артеріальна гіпотензія,

прогресуюча гіперпігментація шкіри. Що лежить в

основі цього симптомокомплексу?

A. * Хронічна гіпофункція кори наднирників

B. Гостра гіпофункція кори наднирників

C. Хронічна гіперфункція кори наднирників

D. Гостра гіперфункція кори наднирників

E. Хронічна гіпофункція мозкової речовини наднирни-

ків

40. У хворого відзначаються періодичні напади серце-

биття (пароксизми), сильне потовиділення, напади

головного болю. При обстеженні виявлена гіпертензія,

гіперглікемія, підвищення основного обміну, тахікардія. 219

При якій патології надниркових залоз спостерігається

подібна картина?

A. *Гіперфункція мозкового шару

B. Гіпофукція мозкового шару

C. Гіперфункція кори наднирків

D. Гіпофункція кори наднирків

E. Первинний альдостеронізм

41. Пацієнт 16 років, який страждає хворобою Іценко-

Кушинга, консультований з приводу надмірної ваги

тіла. Під час дослідження з'ясувалося, що енергетична

цінність споживаної їжі складає 1700-1900 ккал/добу.

Яка провідна причина ожиріння в даному випадку?

A. *Надлишок глюкокортикоїдів

B. Нестача інсуліну

C. Надлишок інсуліну

D. Нестача глюкокортикоїдів

E. Гіподинамія

42. У дитини 6 років розвинулася гіперергічна форма

запалення верхніх дихальних шляхів. З’явилася загро-

за серйозного порушення дихання, а тому виникла

необхідність застосувати протизапальні гормони.

Серед гормонів протизапальний ефект проявляє:

A. *Глюкокортикоїди

B. Катехоламіни

C. Соматотропін

D. Естрогени

E. Мінералокортикоїди

43. У хворого з хронічною серцевою недостатністю

спостерігаються набряки нижніх кінцівок. Надмірна

активація якої системи є головною ланкою їх патоге-

незу?

A. *Ренін-ангіотензин-альдостеронової

B. Гіпоталамо-гіпофізарної

C. Симпато-адреналової

D. Парасимпатичної

E. Калекреїн-кінінової

44. У хворого, який отримав тривалий курс лікування

глюкокортикоїдами, виявлені виразки в шлунку. Який

механізм є головним в їх розвитку?

A. *Збільшення секреції та кислотності шлункового

соку

B. Зниження гістаміну в слизовій оболонці шлунку

C. Підвищення тонусу симпатичної нервової системи

D. Збільшення продукції простагландинів Е1, Е2

E. Зниження тонусу парасимпатичної нервової систе-

ми

45. Хворий звернувся до лікаря зі скаргами на болі у

голові, зміни у кінцівках, збільшення рук та ніг.

Об’єктивно: масивні надбрівні дуги, губи. При надлиш-

ку якого гормону виявляються подібні симптоми?

A. *Соматотропного

B. Адренокортикотропного

C. Тироксину

D. Глюкокортикоїдів

E. Адреналіну

46. Хворий Р., 40 років пред’являє скарги на сильне

серцебиття, пітливість, нудоту, порушення зору, тре-

мор рук, підвищення артеріального тиску. З анамнезу:

2 роки тому було встановлено діагноз феохромоцито-

ма. Гіперпродукція яких гормонів обумовлює дану

патологію?

A. * Катехоламінів

B. Альдостерону

C. Глюкокортикоїдів

D. АКТГ

E. Тиреоїдних гормонів

47. Хворий Д., 45 років поступив в ендокринологічне

відділення з скаргами на головний біль, спрагу, нікту-

рію, періодичні напади судом, підвищення артеріаль-

ного тиску. 6 міс. тому було встановлено діагноз пер-

винний гіперальдостеронізм (хвороба Кона). Гіперпро-

дукція яких гормонів обумовлює дану патологію?

A. *Альдостерону

B. Катехоламінів

C. Глюкокортикоїдів

D. АКТГ

E. Тиреоїдних гормонів

48. Хворий Л., 27 років з діагнозом хвороби Іценко-

Кушинга скаржиться на м’язову слабість, диспептичні

розлади. Об’єктивно: шкіра суха, стоншена, мармуро-

вого відтінку, стрії, АТ 180/140 мм рт.ст. Гіперпродукція

яких гормонів обумовлює дану патологію?

A. *АКТГ

B. Альдостерону

C. Катехоламінів

D. Інсуліну

E. Тироксину

49. Хвора 44 років скаржиться на загальну слабість,

збільшення маси тіла, ріст волосся на обличчі, зупинку

менструацій, АТ 165/100 мм.рт.ст. Що допоможе ди-

ференціювати хворобу Іценко-Кушинга від синдрому

Іценко–Кушинга?

A. *Рівень кортикотропіну в плазмі крові.

B. Рівень кортизолу в плазмі.

C. Вміст 17 –оксикетостероїдів у сечі.

D. Ренгенографія черепу.

E. Кількість еозинофілів в крові.

50. У хворого на аденому клубочкової зони кори над-

нирників (хвороба Конна) спостерігаються артеріальна

гіпертензія, напади судом, поліурія. Що є головною

ланкою в патогенезі симтомів?

A. *Гіперальдостеронізм

B. Гіпоальдостеронізм

C. Гіперсекреція катехоламінів

D. Гіперсекреція глюкокортикоїдів

E. Гіпосекреція глюкокортикоїдів

51. У собаки була змодельовано артеріальну гіпертен-

зію шляхом звуження ниркових артерій. При цьому

збільшилась активність ренін-ангеотензин-

альдостеронової системи. Який компонент цієї систе-

ми викликає найсильніший пресорний ефект?

A. *Ангіотензин ІІ.

B. Ренін.

C. Ангеотензин І.

D. Ангіотензин ІІІ.

E. Альдостерон

52. Хворому на інсулінзалежний цукровий діабет був

введений інсулін. Через деякий час у хворого

з’явилися слабість, дратівливість, посилення потови-

ділення. Який основний механізм розвитку гіпоглікемі-

чної коми, що виникла?

A. *Вуглеводне голодування головного мозку.

B. Посилення глікогенолізу.

C. Посилення кетогенезу.

D. Посилення ліпогенезу.

E. Зменшення гліконеогенезу.

53. У клініку доставили хворого в несвідомому стані, з

рота – запах ацетону. Цукор крові - 25 ммоль/л, кето-

нові тіла –0,57 ммоль/л. При недостатності якого

гормону може розвинутися такий стан?

A. *Інсуліну

B. Тироксину

C. Глюкокортикоїдів

D. Альдостерону

E. Соматотропного гормону

54. На прийом до лікаря прийшов пацієнт дуже високо-

го зросту, з довгими товстими пальцями рук, великою

нижньою щелепою і відвислою нижньою губою. Під-

вищену секрецію якого гормону можна підозрювати?

A. * Соматотропного

B. Тироксину

C. Гонадотропного

D. Альдостерону

E. Катехоламінів

55. У хворої 43 років після чергового загострення

ревмокардиту з’явилися ознаки декомпенсації серце-

вої діяльності з виникненням набряків на ногах і асци-

ту. Затримці води в організмі хворої сприяло підви-

щення продукції

A. *Альдостерону

B. Інсуліну

C. Кортизолу

D. Тироксину

E. Кортикотропіну

56. У хворого виявлено порушення прохідності диха-

льних шляхів на рівні дрібних і середніх бронхів. Які

зміни кислотно-лужної рівноваги можуть розвинутись у

пацієнта?

A. *Респіраторний ацидоз

B. Респіраторний алкалоз

C. Метаболічний ацидоз

D. Метаболічний алкалоз

E. КЛР не зміниться

57. У дитини, хворої на дифтерію, розвинувся набряк

гортані. Який розлад дихання спостерігається у дано-

му випадку?

A. *Диспное (задишка)

B. Гаспінг-дихання

C. Апнеїстичне дихання

D. Дихання Куссмауля

E. Дихання Біота

58. У хворого виявлено ожиріння, гірсутизм, "місяце-

подібне" обличчя, рубці багрового кольору на шкірі

стегон. АТ 180/110 мм рт.ст., глюкоза крові-17,2

ммоль/л. При якій зміні продукції гормонів наднирників

можлива така картина?

A. *Гіперпродукції глюкокортикоїдів

B. Гіпопродукції глюкокортикоїдів

C. Гіперпродукції мінералокортикоїдів

D. Гіпопродукції мінералокортикоїдів

E. Гіпопродукції адреналіну

59. Хворий Д., 35 років, скаржиться на постійну спрагу,

понижений апетит. Кількість рідини, що випивається,

за добу 9 л. Добовий діурез збільшений, сеча знебар-

влена, відносна щільність - 1005. Найбільш вірогідною

причиною розвитку даної патології у хворого є пошко-

дження:

A. *Гіпоталамічних ядер

B. Епітелію ниркових канальців

C. Аденогіпофізу

D. Епіфізу

E. Базальної мембрани капілярів клубочків

60. У хворого В, 46 років, виявлено непропорційне

збільшення кистей рук, стоп ніг, носа, вух, надбрівних

дуг і виличних кісток. У крові - гіперглікемія, порушення

тесту толерантності до глюкози. Причиною розвитку

даної патології швидше за все є:

A. *Гіперсекреція соматотропного гормону

B. Гіперсекреція всіх гормонів аденогіпофізу

C. Гіпосекреція інсуліну

D. Гіпосекреція вазопресину

E. Гіперсекреція глюкокортикоїдів

61. З метою пригнічення аутоімунних реакцій після

пересадки органів обов’язковим єпроведення курсу

гормонотерапі .. Якігормони застосовують з ці .ю ме-

тою?

A. *Глюкокортикоїди.

B. Мінералокортикоїди.

C. Статеві гормони.

D. Адреналін.

E. Соматотропний гормон.

62. У хворого виявлена аденома, що була схожою з

клітинами клубочкової зони кори наднирників. У ре-

зультаті цього розвинувся первинний гіперальдосте-

ронізм або хвороба Кона. На обмін якого іону впливає

цей гормон?

A. * Натрію

B. Хлору

C. Магнію

D. Кальцію

E. Заліза

63. У хворого встановлено стабільне підвищення

температури тіла, тахікардія, емоційна лабільність,

тремор. Зі зміною продукції якого гормону пов’язане

виникнення цього стану?

A. * Тироксину.

B. Вазопресину.

C. Тестостерону.

D. Альдостерону.

E. Інсуліну.

64. У хворого із запальним процесом шкіри і підшкірної

клітковини хронічного перебігу виявлено переважання

процесів проліферації. Нестача якого гормону може

привести до цього?

A. *Кортизон

B. Альдостерон

C. Інсулін

D. СТГ

E. Тироксин

65. Тривале вживання мінералокортикоїдів привело до

появи у пацієнта м’язової слабкості. Що лежить в

основі патогенезу цього явища?

A. *Гіпокаліємія

B. Гіперкаліємія

C. Гіпернатріємія

D. Гіпонатріємія

E. Гіперволемія

66. У хворого Н., 25 років, після перенесеної інфекції

розвинувся нецукровий діабет. Дефіцит якого гормону

привів до розвитку даної патології?

A. *Вазопресину

B. Альдостерону

C. Кортизолу

D. Реніну

E. Інсуліну

67. Хворий Д., 50 років було встановлено діагноз

мікседема. Порушення утворення яких гормонів при-

водить до розвитку даної патології?

A. *Тироксину і трийодтироніну

B. Кортизолу і альдостерону

C. АКТГ і СТГ__220

D. Окситоцину і вазопресину

E. Інсуліну і глюкагону

68. У хворого на гіпертонічну хворобу виявлено в крові

збільшення концентрації вазопресину. На функцію

якого органу впливає цей гормон?

A. *Нирок

B. Печінки

C. Серця

D. Легень

E. Наднирників

69. У дівчинки діагностовано адреногенітальний синд-

ром (псевдогермофродитизм). Надмірна секреція

якого гормону наднирників обумовило дану патологію?

A. *Андрогенів

B. Естрогенів

C. Альдостерону

D. Кортизолу

E. Адреналіну

70. При клінічному обстеженні в жінки встановлено:

підвищенне потовиділення, тахікардія, схуднення,

тремор. Яка ендокринна патологія може це спричини-

ти?

A. * Гіпертиреоз

B. Гітотиреоз

C. Гіпергонадизм

D. Гіпогонадизм

E. Гіпоальдостеронізм

71. Ріст дитини 10 років досягає 178 см, маса – 64 кг З

порушенням діяльності якої ендокринної залози це

пов’язано?

A. * Гіпофізу

B. Щитовидної залози

C. Статевих залоз

D. Надниркових залоз

E. Паращитоподібної залози

72. Після перенесеного сепсису у хворої 27 років

з’явився бронзовий колір шкіри, характерний для

Аддісонової хвороби. У підвищенні секреції якого

гормону полягає механізм гіперпігментації?

A. *Меланоцитстимулюючого

B. Соматотропного

C. Гонадотропного

D. B-ліпотропного

E. Тиреотропного

73. Внаслідок травми у хворого були пошкоджені

нервові шляхи, які проходять крізь ніжку гіпофіза.

Надходження в кров яких гормонів буде порушено?

A * Вазопресину

B Соматотропіну

C Тиреотропіну

D Пролактину

E Адренокортикотропіну

74. У хворого внаслідок черепно-мозкової травми

з’явилась значна поліурія, що не супроводжувалася

глюкозурією. Пошкодження якої структури може при-

звести до подібних змін?

A *Задньої частки гіпофізу.

B Передньої частки гіпофізу.

C Мозкової речовини наднирників.

D Кори наднирників.

E Підшлункової залози.

75. У чоловіка, віком 60 років, встановлено синдром

Іценко-Кушинга. Про що свідчить наявність негативно-

го азотистого балансу у цього хворого?

A *Про посилення катаболічних процесів в клітинах.

B Про посилення анаболічних процесів в клітинах.

C Про посилення утворення глікогену в клітинах

печінки.

D Про підвищення чутливості клітин до

катехоламінів.

E Про посилення клубочкової фільтрації в нирках.

76. В дослідах на щурах, при моделюванні гравітацій-

ного стресу, виявлено однотипні зміни. Що буде

специфічною ознакою для стресу?

A *Інволюція тиміко-лімфатичного апарату

B Атрофія кори наднирників

C Гіперемія селезінки

D Гранулоцитопенія

E Гіпотрофія кори наднирників

77. У тварини, що знаходиться в стані стресу, спосте-

рігаються клінічні ознаки загального адаптаційного

синдрому (гіпотонія м’язів та артеріальна гіпотензія,

гіпотермія, гіпоглікемія, згущення крові, еозинопенія).

Яка стадія стресу у тварини?

A * Фаза шоку

B Фаза протишоку

C Стадія виснаження

D Стадія резистентності

E Торпідна стадія

78. При обстеженні у хворого виявлено: м’язеву

слабість, зниження артеріального тиску, розлади

сприйняття смаку, запахів, звуків. Що може бути при-

чиною цих проявів?

A *Недостатня продукція глюкокортикоїдів

B Гіпофункція щитовидної залози

C Гіпофункція статевих залоз

D Гіпофункція мозкового шару наднирників

E Гіпофункція прещитовидних залоз

79. У хворого з синдромом Іценко-Кушинга спостеріга-

ється стійка гіперглікемія та глюкозурія. Синтез та

секреція якого гормону збільшується в даному випад-

ку?

A Кортизолу

B Адреналіну

C Глюкагону

D Тироксину

E Альдостерону

80. Хлопчик віком 5 місяців госпіталізований з приводу

тонічних судом. Хворіє з народження. При огляді

волосся жорстке, нігті витончені та ламкі, шкіряні

покриви бліді та сухі., в біохімічному аналізі крові

кальцій 1,5 Ммоль/л, фосфор – 1,9 Ммоль/л. Який

найбільш ймовірний діагноз?

A * Гіпопаратиреоз

B Гіперпаратиреоз

C Гіперальдостеронізм

D Гіпоальдостеронізм

E Гіпотиреоз

81. Пацієнта було прооперовано з приводу гіперплазії

щитоподібної залози. Після операції з’явилися судоми.

Лабораторне дослідження виявило зниження кальцію

та фосфору. Що спричинило таку ситуацію?

A * Під час операції було ушкоджено паращитоподібну

залозу, яка продукує гормон, що регулює вміст

кальцію в крові

B Резекція частини щитоподібної залози спричинила

зниження вмісту кальцію в крові

C Під час операції було ушкоджено гортань

D Резекція частини щитоподібної залози сприяє

розвитку тіреотоксикозу

E Резекція частини щитоподібної залози сприяє роз-

витку діабету

82. Жінка, 42 років, звернулась зі скаргами на знижен-

ня маси тіла, посилення пітливості, субфібрилітет,

дратівливість. Підвищення секреції якого гормону

може викликати ці явища?

A Тироксина

B Соматотропіна

C Кортикотропіна

D Інсуліна

E Альдостерона

83. У хворого - двобічна гіперплазія кори наднирників.

При обстеженні мозку яка ділянка буде цікавити?

A * Гіпофіз

B Мозочок

C Скронева ділянка головного мозку

D Епіфіз

E Стовбур мозку

84. Під час клінічного обстеження пацієнтки виявлено:

зниження основного обміну на 40 %, збільшення маси

тіла, зниження температури тіла, одутловатість об-

личчя, порушення статевих функцій, в’ялість та апатія,

зниження інтелекту. Яка ендокринна залоза уражена?

A * Гіпофункція щитовидної залози

B Гіпофункція паращитовидної залози

C Гіперфункція гіпофізу

D Гіпофункція епіфізу

E Гіперфункція щитовидної залози

85. Під час клінічного обстеження пацієнтки виявлено:

збільшення щитовидної залози (зоб), підвищення

основного обміну, втрата ваги тіла, надмірну пітли-

вість, підвищений апетит, дратівливість, екзофтальм

та тахікардію. Яка ендокринна залоза уражена?

A * Гіперфункція щитовидної залози

B Гіпофункція паращитовидної залози

C Гіперфункція гіпофіза

D Гіпофункція епіфізу

E Гіпофункція щитовидної залози

86. Хворий з феохромоцитомою скаржиться на спрагу,

сухість в роті, підвищення апетиту. Глюкоза в крові 8,7

ммоль/л. Яке походження гіперглікемії:

A * Адреналова

B Гіперкортикоїдна

C Аліментарна

D Соматотропна

E Гіпоінсулінемічна

87. До лікаря звернулась жінка 58 років, зі скаргами на

прогресуюче збільшення ваги тіла, підвищену втом-

люваність; із анамнезу відомо, що клімакс наступив в

55 років, харчування помірне. Чим зумовлене ожирін-

ня в жінки?

A * Гіпофункція яйників

B Хвороба Іценко-Кушинга

C Аліментарний дисбаланс

D Пухлина яйника

E Гіпофункція щитовидної залози

88. У хворого з аддісоновою хворобою після фізичного

навантаження виникли відчуття нестерпного голоду,

тахікардія, слабкість, пітливість. Зниження синтезу і

недостатність яких ферментів є механізмом розвитку

гіпоглікемії у хворого?

A * Глюконеогенезу

B Глікогенолізу

C Глікогенезу

D Гліколізу

E Гексокінази

89. У хворої з тиреотоксикозом спостерігається

роз’єднання окисного фосфорилювання. До яких змін

це призводить?

A * Збільшення теплоутворення

B Зменшення частки вільного окислення

C Зменшення теплоутворення

D Теплоутворення не змінюється

E Збільшення утворення АТФ

90. У хворого К., 42 р. виявили пухлину аденогіпофізу.

Об’єктивно: вага 117 кг, обличчя місяцеподібне,

гіперемоване, на шкірі живота синьо-багряні смуги

розтягу. Остеопороз, дистрофія м’язів. АТ 210/140 мм

рт. ст У хворого найбільш імовірно:

A * Хвороба Іценка-Кушинга

B Синдром Іценка-Кушинга

C Хвороба Конна

D Цукровий діабет

E Гіпертонічна хвороба

91. У хворого з печінковою комою в крові підвищилась

концентрація Na+ та знизилась концентрація К+, тобто

почались прояви гіперальдостеронізму. Чим зумовле-

но підвищення дії альдостерону у хворого?

A *Зменшенням його метаболізму.

B Підвищенням синтезу альдостерону.

C Збільшенням кількості рецепторів до нього.

D Порушенням його транспорту.

E Зменьшенням його виведення.

92. Хвора звернулася до лікаря зі скаргами на схуд-

нення, швидку фізичну і психічну втомлюваність, пога-

ний апетит, зміну кольору шкіри, артеріальну гіпотен-

зію. При огляді виявлено гіперпігментацію шкіри. Її

механізм пов’язаний з:

A *Посиленням меланоцитстимулюючої активності

гіпофіза

B Зниженням меланоцитстимулюючої активності

гіпофіза

C Зниженням рівня адреналіну в крові

D Переважанням катаболізму білків

E Підвищенням концентрації іонів натрію

93. Хворий звернувся до лікаря із скаргами на збіль-

шення розмірів носа, підборіддя, пальців кінцівок. При

об’єктивному обстеженні виявлено: розширення меж

серця, збільшення розмірів печінки, селезінки. Причи-

ною розвитку вказаних порушень є:

A *Підвищення секреції соматотропного гормону

B Підвищення секреції тиреотропного гормону

C Підвищення секреції адренокортикотропного гор-

мону

D Підвищення секреції альдостерону

E Підвищення секреції адреналіну

94. Чому у деяких географічних регіонах України кіль-

кість людей з гіпертрофією щитовидної залози значно

більша ніж в інших регіонах?

A * Низький рівень йоду у питній воді викликає компен-

саторну реакцію залози – її розростання.

B Високий рівень кальцію у воді стимулює ріст щито-

видної залози.

C Низький рівень кальцію у воді стимулює ріст

щитовидної залози.

D Радіоактивний фон стимулює ріст щитовидної

залози.

E Високий рівень йоду у воді стимулює ріст щитовидної

залози.

95. Юнак 17-ти років скаржиться на порушення сну,

зниження маси тіла, серцебиття. Після обстеження__221

встановлено гіперплазію щитовидної залози ІІ ступе-

ню. Які порушення рівнів гормонів найбільш характерні

для захворювання?

A * Підвищення тироксину

B Зниження тироксину

C Підвищення соматотропіну

D Зниження соматотропіну

E Зниження трийодтироксину

96. Тетанія – синдром підвищеної нервової

збудливості, проявляється розвитком судом за умов

гіпокалійемії. Який патогенетичний механізм є

провідним у даному випадку?

A *Зменшення порогу збудливості клітин

B Збільшення порогу збудливості клітин

C Порушення функціонування натрій-калієвих

насосів

D Зниження синтезу АПФ у клітинах

E Розвиток ацидозу

97. При обстеженні у пацієнта виявлена недостатня

продукція глюкокортикоїдів. Які симптомокомплекси

можуть бути при цьому?

A *Зниження артеріального тиску, м’язева слабкість,

розлади сприйняття смаку, запахів і звуків

B Підвищення кров’яного тиску

C Лімфопенія

D Еозинопенія

E Лімфоцитоз

98. У пацієнта виявлено пухлину правого наднирника.

Який симптомокомплекс буде при цьому?

A *Периферичного синдрому Іценко-Кушинга

B Базедової хвороби

C Гіпертиреозу

D Ниркової недостатності

E Аддісонової хвороби

99. У хворої при обстеженні виявлено гірсутизм, "міся-

цеподібне обличчя", смуги розтяжок на шкірі живота.

Артеріальний тиск 190/100 мм рт.ст., глюкоза крові -

17,6 ммоль/л. При якому з вказаних видів патології

зустрічається подібна картина?

A.*Гіперфункції кори наднирників

B. Гіпертиреозі

C. Гіпотиреозі

D. Гіпофункції статевих залоз

E. Гіперфункції інсулярного апарату

100.Хворому тривалий час вводили високі дози

гідрокортизону, внаслідок чого настала атрофія однієї

з зон кори наднирників. Яка це зона?

A. *Пучкова

B. Сітчаста

C. Клубочкова

D. -

E. Клубочкова і сітчаста

101. Тварині внутрішньовенно ввели концентрований

розчин хлориду натрію, що зумовило зниження

реабсорбції іонів натрію у канальцях нирок. Внаслідок

яких змін секреції гормонів це відбувається?

A. *Зменшення альдостерону

B. Збільшення вазопресину

C. Збільшення альдостерону

D. Зменшення натрійуретичного фактора

E. Зменшення вазопресину

102. Пацієнта турбують поліурія (7л на добу) і

полідипсія. При обстеженні не виявлено ніяких

розладів вуглеводного обміну. Дисфункція якої

ендокринної залози може бути причиною даних пору-

шень?

A. *Нейрогіпофіз

B. Острівці підшлункової залози

C. Аденогіпофіз

D. Кора наднирників

E. Мозкова речовина наднирників

103. Хлопчик 5-ти місяців госпіталізований з приводу

тонічних судом. Хворіє з народження. Об'єктивно:

волосся жорстке, нігті витончені та ламкі, шкірні по-

криви бліді та сухі. В біохімічному аналізі крові: кальцій

- 0,5 ммоль/л (норма - 0,75-2,5 ммоль/л), фосфор - 1,9

ммоль/л (норма - 0,646-1,292 ммоль/л). З чим

пов'язані ці зміни?

A. *Гіпопаратиреоз

B. Гіперальдостеронізм

C. Гіперпаратиреоз

D. Гіпотиреоз

E. Гіпоальдостеронізм

104. Тварині внутрішньовенно ввели концентрований

розчин хлориду натрію, що зумовило зниження

реабсорбції іонів натрію у канальцях нирок. Внаслідок

яких змін секреції гормонів це відбувається?

A. *Зменшення альдостерону

B. Зменшення натрійуретичного фактора

C. Збільшення альдостерону

D. Збільшення вазопресину

E. Зменшення вазопресину

Патофізіологія нервової системи

1. Хворий 32 років поступив в стаціонар зі скаргами на

загальне недомагання, нудоту, біль справа внизу

живота. Після огляду лікарем було встановлено діаг-

ноз – гострий апендицит. Який вид болю у хворого?

A. *Вісцеральний

B. Соматичний глибокий.

C. Соматичний поверхневий ранній

D. Соматичний поверхневий пізній.

E. Фантомний

2. В результаті перенесеної нейроінфекції хворий

втратив здатність визначати напомацки знайомий

предмет. Як називається порушений вид чутливості?

A. *Стереогноз

B. Відчуття дискримінації

C. Двовимірно-просторове відчуття

D. Відчуття локалізації

E. М’язово-суглобове відчуття

3. У хворого Н., 49 років, відмічається обмеження

довільних рухів в лівих кінцівках. Тонус м’язів в лівій

руці і нозі підвищений за спастичним типом, посилені

місцеві сухожильні рефлекси, виявляються патологічні

рефлекси. Який найбільш вирогідний механізм привів

до розвитку м’язової гіпертонії і гіперрефлексії?

A. *Зниження гальмівних низхідних впливів

B. Активація мотонейронів внаслідок інсульту

C. Активація збудливих впливів з вогнища інсульту

D. Активація синаптичної передачі імпульсів

E. Гальмування мотонейронів кори головного мозку

4. У хворого Л., 40 років, об’єм м’язів в області гомілки

правої ноги на 2 см менше, ніж на лівій. Ахіловий і

колінний рефлекси справа відсутні. Який найбільш

вірогідний механізм виникнення гіпорефлексії при

периферичному паралічі?

A. * Порушення проведення збудження

B. Гальмування пірамідних мотонейронів

C. Порушення синаптичної передачі імпульсів

D. Активація збудливих впливів з ЦНС

E. Порушення сприйняття збудження

5. Після автомобільної катастрофи у хворого діагнос-

товано травму середньої третини плеча з неповним

розривом серединного нерва. Крім рухових та сенсор-

них розладів нижче місця травми хворий скаржиться

на різкі, пекучі, нестерпні болі. Якого характеру є ці

болі?

A. *Каузалгія

B. Проекційний біль

C. Відбитий

D. Фантомний

E. Соматичний

6. У хворого внаслідок травми поперекового відділу

хребта відбувся розрив половини спинного мозку. При

обстеженні на боці ураження має місце параліч кінців-

ки, втрата тактильної та глибокої суглобово-м’язової чутливості на протилежному боці- втрата больовоїі температурноїчутливості Для якого синдрому харак-

терний такий симптомокомплекс?

A. *Броун-Секара

B. Верніке-Корсакова

C. Джуліана-Барра

D. Ламберта-Ітона

E. Паркінсона

7. У експериментальної тварини видалили мозочок. В

результаті рухи втратили плавність, точність, зникла

відповідність рухів. Як називається описане явище?

A. *Атаксія

B. Гіперкінез

C. Тремор

D. Ригідність

E. Атетоз

8. В експерименті видалили частину головного мозку,

внаслідок чого в тварини розвинулись асинергія і

дисметрія. Який відділ головного мозку був видалений

у тварини?

A. * Мозочок

B. Лобна частка

C. Тім’яна частка

D. Середній мозок

E. Сітчастий утвір

9. У хворих з В12 - дефіцитною анемією виникають

дегенеративні процеси в задніх і бокових стовпах

спинного мозку (фунікулярний мієлоз). Ураження

аксонів пов’язане з порушенням утворення:

A. *Мі .ліну

B. Ацетилхоліну

C. Дофаміну

D. Серотоніну

E. Норадреналіну

10. Після ампутації кінцівки хворий тривалий час від-

чуває кінцівку та сильний, нестерпний біль у ній. Як

називається такий біль?

A. * Фантомний

B. Рефлекторний (відбитий)

C. Вісцеральний

D. Первинний (протопатичний )

E. Вторинний (епікритичний)

11. У хворого діагностовано крововилив у внутрішню

капсулу. У результаті були пошкоджені моторні

провідні шляхи з одного боку. Які зміни це спричинить

в організмі?

A *Спастичний параліч на протилежній стороні

B Спастичний параліч на даній стороні

C Втрата відчуття дотику та тиску на протилежній

стороні

D Млявий параліч на даній стороні

E Млявий параліч на протилежній стороні

12. В експерименті показано, що при больовій

стимуляції тварини активуються енкефалінергічні

нейрони з виділенням анкефалінів. До якої

антиноцицепивної (анальгетичної) системи можуть

відноситися енкефалінергічні нейрони?

A *Нейронна опіатна система.

B Гормональна опіатна система.

C Нейронна неопіатна система.

D Гормональна неопіатна система.

E -.

13. Травма спинного мозку призвела до паралічу

справа і порушення больової та температурної

чутливості зліва. Де локалізується пошкодження спин-

ного мозку?

A * Одностороннє пошкодження спинного мозку з

правої сторони

B Одностороннє пошкодження спинного мозку з лівої

сторони

C Пошкодження спинного мозку з обох сторін

D Пошкодження задніх корінців

E Пошкодження передніх корінців

14. Травма головного мозку привела до порушення

тонких рухів, м’язевої ригідності і тремору. Де

локалізується пошкодження головного мозку?

A *Пошкодження середнього мозку в ділянці чорної

субстанції

B Порушення функції спинного мозку

C Пошкодження рухової кори

D Пошкодження лімбічної системи

E Пошкодження стовбуру мозку

15. В неврологічне відділення поступив хворий з трав-

мою шийного відділу хребта. Об’єктивно: зникнення

рефлексів нижче місця ушкодження. Який механізм

розвитку?

A * Відсутність кортико-спінального контролю

B Ураження спинно-мозочкових шляхів

C Ураження спінальних рефлекторних дуг

D Вітсутність ретикуло-спінальних впливів

E Ураження спіноталамічних шляхів

16. Після автомобільної катастрофи у хворого ура-

ження спинного мозку на рівні 3-4 шийних хребців. Як

зміниться дихання?

A * Відбудеться зупинка дихання

B Тахіпноє

C Брадипноє

D Збільшиться пауза між вдихом та видихом

E Дихання не зміниться

17. Гіпоксичне ушкодження півкуль мозочка призвело

до порушення рухових функцій. Як будуть проявляти-

ся ці порушення?

A * Неможливість швидких цілеспрямованих рухів

B Гіпертонус

C Гіпотонус

D Хорея

E Атетоз

18. У жінки після геморагічнгого інсульту порушена

рухова функція правої верхньої та нижньої кінцівок. В

якій ділянці кори відбувся крововилив?

A * Передня центральна звивина зліва

B Передня центральна звивина справа

C Задня центральна звивина справа__222

D Задня центральна звивина зліва

E Потилична доля кори мозку

19. У дорослої людини на фоні психо-емоційного

стресу збільшилась збутливість кори головного мозку.

Який механізм змін?

A *Збільшення тонусу ядер ретикулярной формації

стовбура мозку

B Збільшення тонусу червоних ядер

C Збільшення тонусу базальних гангліїв

D Збільшення тонусу лімбічної системи

E Збільшення тонусу чорної субстанції

20. У хворого спостерігається різке підвищення тонусу

розгинальних м’язів, утруднення рухів. Порушення

діяльності нейронів якого відділу мозку є причиною

децеребраційної ригідності в хворого?

A * Стовбурної частини

B Кори

C Мозочка

D Гіпоталамуса

E Спинного мозку

21. У чоловіка віком 30 років після струсу головного

мозку виникла блювота. Який патогенетичний варіант

блювоти спостерігається у цьому випадку?

A *Центральний, під впливом механічної травми.

B Центральний, під впливом активації рецепторів

лабіринту.

C Вісцеральний, під впливом підразнення печінки і

нирок.

D Гематогенно-токсичний, під впливом токсинів із

крові.

E Рефлекторний, під впливом підразнення блукаючого

нерва шлунку.

22. У хворого, що перебуває в стані коми, розвинувся

набряк головного мозку з прогресуючими розладами

серцевої і дихальної функцій. Ураження якого відділу

головного мозку призвело до життєво небезпечних

процесів?

A *Довгастий мозок

B Мозочок

C Середній мозок

D Проміжний мозок

E Кора головного мозку

23. Яке електричне явище можна виявити у м’язевій

клітині на першій стадії після денервації, поки

бездіяльність не призведе до її атрофії?

A *Спонтанні потенціали дії (потенціали фібриляції).

B Локальний потенціал.

C Потенціал пошкодження.

D Пасивну деполяризацію.

E Гіперполяризацію.

24. Хворий звернувся до невропатолога із скаргами на

порушення ходи, рівноваги тіла, тремор тулуба і

кінцівок, дискоординацію рухів, зміну почерку. У

патогенезі наявних неврологічних розладів

найімовірніше має значення порушення функції:

A * Мозочка

B Стовбура головного мозку

C Екстрапірамідної системи

D Кори головного мозку

E Лімбічної системи

25. На прийомі у лікаря хворий раптово втратив

свідомість, через хвилину у нього розвинулись судоми,

які мали характер короткочасних скорочень та розс-

лаблень окремих груп м’язів, що швидко слідують

одне за одним. Який вид судомів розвинувся у хворо-

го?

A * Клонічні

B Тонічні

C Тоніко-клонічні

D Атетоз

E Гемібалізм

26. У жінки 52 років артеріальна гіпертензія ускладни-

лась правобічною геміплегією і втратою мови. Яка

зона головного мозку є найбільш ймовірно ураженою?

A * Ліва передня звивина і ліва скронева доля головно-

го мозку

B Ліва передня звивина головного мозку.

C Ліва скронева доля мозку.

D Права передня звивина головного мозку

E Потилична частка головного мозку

27. В експерименті у кішки після перерізки спинного

мозку виник спінальний шок. Об’єктивно: нижче місця

перерізки відсутня рухова рефлекторна активність.

Через який термін рухова активність відновиться?

A * Через 2-3 тижні.

B Через 15 хв.

C Через 1 годину.

D Через 5 хв.

E Ніколи не відновиться.

28. Підвищення тактильної та больової чутливості

шкірних покривів при захворюванні внутрішніх органів

виникає внаслідок:

A *Сегментарної будови автономної та соматичної

інервації ділянок шкіри

B Ураження сомато-сенсорних зон кори головного

мозку

C Ураження средніх ядер гіпоталамусу

D Ураження таламічної ділянки головного мозку

E Ураження задніх канатиків спинного мозку

29. Під час автопригоди у людини виникло

одностороннє пошкодження спинного мозку з правої

сторони. Які симптомокомплекси спостерігались при

цьому?

A *Паралічі, розлади больової чутливості справа; зліва

– порушення

температурної і больової чутливості зліва

B Порушення больової чутливості справа

C Порушення тактильної чутливості справа

D Порушення суглобовомязеваго відчуття зліва

E Паралічі зліва

30. В експерименті у тварини зруйнували середній

мозок в ділянці чорної субстанції. Синтез якого медіа-

тора припинився?

A *Дофаміну

B Ацетилхоліну

C Серотоніну

D Гістаміну

E Норадреналіну

31. Що лежить в основі механізму формування пато-

логічної детермінанти?

A.*Формування генератора патологічно посиленого

збудження.

B. Феномен випадіння

C. Позамежове гальмування

D. Перезбудження

E. Парабіоз

32. Після перенесеного сепсису у хворої 27-ми років

з'явився бронзовий колір шкіри, характерний для

аддісонової хвороби. Механізм гіперпігментації поля-

гає в підвищенні секреції такого гормону:

A. *Меланоцитстимулюючий

B. B-ліпотропний

C. Соматотропний

D. Гонадотропний

E. Тиреотропний

ПАТОМОРФОЛОГІЯ

Патологія клітини. Дистрофії. Некроз.

Смерть

1. При черевному тифі некротизовані пейєрові бляшки

тонкої кишки забарвлюються у жовто-коричневий

колір. Який пігмент просякає некротизовану тканину?

A. * Білірубін

B. Гемоглобін

C. Ліпофусцин

D. Індол

E. Меланін

2. Хвора 70 років прооперована з приводу “гострого

живота”. Під час операції виявлено близько 80 см

клубової кишки чорного кольору, очеревина тьмяна,

просвіт верхньої брижової артерії обтурований тром-

бом. Який процес розвинувся в кишці?

A. *Гангрена

B. Пролежень

C. Білий інфаркт

D. Білий інфаркт із геморагічним вінчиком

E. Коагуляційний некроз

3. На розтині трупа на підставі характерних

макроскопічних змін діагностована паренхіматозна

жирова дистрофія міокарда серця. Яка образна назва

серця при цій дистрофії?

A. *Тигрове серце

B. Волове серце

C. Волосате серце

D. Панцирне серце

E. Легеневе серце

4. У хворого цукровим діабетом з'явився різкий біль у

правій стопі. При огляді великий палець стопи чорного

кольору, тканини стопи набряклі, осередки

відшарування епідермісу, виділення з неприємним

запахом. Яка клініко-морфологічна форма некрозу

розвилася в хворого?

A. *Гангрена волога

B. Пролежень

C. Секвестр

D. Гангрена суха

E. Інфаркт

5. У хворого з вадою мітрального клапану з'явився

кашель, мокротиння ржавого кольору. Який пігмент

обумовив такий колір мокротиння?

A. *Гемосидерин

B. Меланін

C. Гемоглобін

D. Гемомеланін

E. Сірчасте залізо

6. У хворого виразковою хворобою шлунка з кровоте-

чею при ендоскопії в шлунку знайдена рідина кольору

кавової гущавини. Який пігмент обумовив такий колір

вмісту шлунка?

A. *Солянокислий гематин

B. Гемосидерин

C. Білірубін

D. Феритин

E. Порфірин

7. Жінка віком 68 років хворіє на хронічний фіброзно-

кавернозний туберкульоз легень протягом 20 років.

Поступив у нефрологічне відділення з явищами уремії.

Прижиттєва проба на наявність у нирках амілоїду

виявилася позитивною. Про яку форму амілоїдозу йде

мова в даному випадку?

A. *Вторинний системний

B. Первинний системний

C. Обмежений (місцевий)

D. Сімейний вроджений

E. Сенильний (старечий)

8. На розтині трупа чоловіка віком 57 років, який помер

від висипного тифу, виявлено, що м’язи передньої черевноїстінки і стегон щільні, білуваато-жовтого

кольору, нагадують стеаринову свічу. Проявом якого

патологічного процесу є виявлені зміни?

A. *Восковидного некрозу

B. Апоптозу

C. Фібриноїдного некрозу

D. Колікваційного некрозу

E. Казеозного некрозу

9. У померлого 58 років на розтині мітральний клапан

деформований, потовщаний, змикається не до кінця.

Мікроскопічно: вогнища колагенових волоконець

еозінофільні, дають плюсову реакцію на фібрін.

Найвірогідніше це:

A. *Фібріноїдне набухання

B. Фібрінозне запалення

C. Мукоїдне набухання

D. Гіаліноз

E. Амілоїдоз

10. При розтині чоловіка 49 років, який поступив у

стаціонар з картиною гепатотропної інтоксикації і

раптово помер, печінка збільшена, дрябла, жовто-

коричневого кольору; на поверхні розрізу печінки і лезі

ножа помітні краплини жиру. Мікроскопічно: гепатоци-

ти периферії класичних печінкових часточок вміщують

масу дрібних крапель, які виповнюють цитоплазму і

відсовують ядро на периферію. Який процес

найімовірніше має місце в печінці ?

A. *Жирова дистрофія печінки

B. Цереброзидліпідоз (хвороба Гоше)

C. Сфінгомієлінліпідоз (хвороба Німанна-Піка)

D. Гангліозидліпідоз (хвороба Тея-Сакса)

E. Генералізований гангліозидоз(хвороба Нормана-

Ландінга)

11. У хворого на гостру виразкову хворобу шлунка, що

ускладнилась шлунковою кровотечею, блювотні маси

забарвлені в темно-коричневий колір, що описується

як блювота “кавовою гущею”. Наявність якого пігменту

в блювотних масах визначає таке їх забарвлення?

A. *Солянокислого гематину

B. Гемоглобіну

C. Білірубіну

D. Гемомеланіну

E. Сульфіду заліза

12.У померлого при розтині знайдено тромбоз лівої

середньої мозкової артерії і велике вогнище сірого

пом’якшення тканин лівої півкулі мозку. Який

патологічний процес розвився в головному мозку?

A. * Ішемичний інфаркт

B. Коагуляційний некроз

C. Абсцесс__




1. Стратегические бомбардировщики ВВС США
2. Лабораторная работа 5б ОПРЕДЕЛЕНИЕ ОТНОШЕНИЯ ТЕПЛОЕМКОСТЕЙ CP - CV ДЛЯ ВОЗДУХА МЕТОДОМ ЗВУКО
3. Лабораторная работа 2 Цель- Выполнить перевод десятичных дробей из десятичной системы счисления в двои
4. Безопасность жизнедеятельности СанктПетербург 2000 УДК 614
5. коэффициентов аппроксимирующего полинома
6. Аксиома Проводит МеждународнУЮ научнопрактическУЮ конференциЮ Психология и педагогика- совре.html
7. тематика Направление ~ 120700
8. Секрет вращения земли и причины возникновения циклонов, тайфунов, торнадо
9. Поле для игры и ворота Размеры.html
10. Условия и порядок заключения брака
11. Безпека життєдіяльності Програма Нормативної навчальної дисципліни підготовки бакал
12. Вивчення рівнів вихованості дитини
13.  Установите соответствие между наименованием роли менеджера в организации и ее характеристикой
14. Шпет Скептик и его душаЭтюд по философской интерпретации Мышление уже имеет некоторую историю к
15. тема организационнораспорядительной документации
16. Московский Комсомолец 31
17. Курсовая работа- Психологическая готовность к обучению в школе
18. . вмещающая среда для сооружений оснований сооружений; 2
19. ru Все книги автора Эта же книга в других форматах Приятного чтения Гэри Чепмен Пять языков люб
20. тема физическая система не содержащая частей отличающихся по составу или свойствам и отделённых друг от др